Sie sind auf Seite 1von 100

QUESTION A newborn infant develops respiratory distress immediately

1 of 45
Questions
after birth. His abdomen is scaphoid. No breath sounds are
Notes heard on the left side of his chest, but they are audible on the
right. Immediate intubation is successful with little or no
improvement in clinical status. Emergency chest x-ray is shown
below.

The most likely explanation for this infant’s conditions is which


of the following:

pneumonia

cyanotic heart disease

diaphragmatic hernia

choanal atresia

pneumothorax

Click on image to Zoom

Explanation

The correct answer is choice C.

Congenital diaphragmatic hernia occurs due presence of developmental defects in the


diaphragm through the bowel loops herniates to chest compressing the lungs and producing
pulmonary hypoplasia, pulmonary hypertension, pulmonary immaturity, and potential
deficiencies in the surfactant and antioxidant enzyme system. In 85% of cases the herniation
occurs through a defect on the left side of diaphragm called foramen of Bochdalek. Lung
compression explains the signs seen in this case as respiratory distress and absent breath
sounds on one side. The x-ray show mediastinal shift to the right and air filled bowel in the left
side.
Answer A is false. In pneumonia, auscultation would reveal focal crackles and wheezes and
not absence of breath sounds. X-ray would show the pneumonic patches with no shift of
mediastinum.

Answer B is false as cyanotic heart disease would not produce absent of breath sounds but
murmurs of the cardiac lesions will be heard. Also the x-ray would show the anomalies of the
cardiac shadow.

Answer D is false. Choanal atresia, a complete nasal obstruction in a newborn, can produce
severe respiratory distress but the signs of diminished air entry into the chest would not be
unilateral as in this condition.

Answer E is false. Pneumothorax can produce a picture similar to this condition but the x-ray
in pneumothorax would show jet-black translucency with collapse of the underlying ling.
QUESTION A 75-year-old man from China presents to GI clinic complaining
2 of 45
Questions
of increased abdominal girth, weight loss, and fever for the
Notes past year. He has been increasing nauseous for the past month
to the point that he can not hold down food anymore without
vomitting. Physical exam is remarkable for right upper quadrant
tenderness and marked hepatomegaly. He is visibly jaundiced.
Abdominal ultrasound reveals gall stones and 3.7 cm common
bile duct. His lab results are as follows:

 AST 59 (N<30)
 ALT 37 (N<35);
 ALP 320 (N<120)
 Conjugated bilrubin 70 (N<5);
 Unconjugated bilrubin 10 (N<15)

Which one of the following is most likely to account for his


symptoms?
sclerosing cholangitis

common bile duct stone

adenocarcinoma of the head of the pancreas

acute cholestasis

hepatocellular carcinoma

Explanation

The correct answer is choice E

This patient's symptoms are consistent with a hepatic process. Hepatocellular carcinoma (HCC)
is a primary malignancy of the liver. Most cases of HCC are secondary to either a viral
hepatitide infection (hepatitis B or C) or cirrhosis (alcoholism being the most common cause of
hepatic cirrhosis). Of note, China is an area of endemic Hepatitis B infection.

HCC may present with jaundice, bloating from ascites, easy bruising from blood clotting
abnormalities, loss of appetite, unintentional weight loss, abdominal pain,especially in the
upper -right part, nausea, emesis, or fatigue.

Hepatocellular carcinoma (HCC) most commonly appears in a patient with chronic viral
hepatitis (hepatitis B or hepatitis C, 20%) or with cirrhosis (about 80%). These patients
commonly undergo surveillance with ultrasound due to the cost-effectiveness. In patients with
a higher suspicion of HCC (such as rising alpha-fetoprotein), the best method of diagnosis
involves a CT scan of the abdomen using intravenous contrast agent and three-phase scanning
(before contrast administration, immediately after contrast administration, and again after a
delay) to increase the ability of the radiologist to detect small or subtle tumors. It is important
to optimize the parameters of the CT examination, because the underlying liver disease that
most HCC patients have can make the findings more difficult to appreciate.

Surgical resection offers the best prognosis for long-term survival but only 10-15% of patients
are suitable for surgical resection. This is often due to extensive disease or poor liver function.
Resection in cirrhotic patients carries high morbidity and mortality. The overall recurrent rate
after resection is 50-60%.

Sclerosing cholangitis (choice A) is a chronic liver disease caused by progressive inflammation


and scarring of the bile ducts of the liver. The inflammation impedes the flow of bile to the gut,
which can ultimately lead to liver cirrhosis, liver failure and liver cancer. The underlying cause
of the inflammation is believed to be autoimmunity. The diagnosis is by imaging of the bile
duct, usually in the setting of endoscopic retrograde cholangiopancreatography (ERCP,
endoscopy of the bile duct and pancreas), which shows "beading" (both strictures and dilation)
of the intrahepatic and extrahepatic bile ducts.

Choledocholithiasis (choice B) is the presence of gallstones in the common bile duct. This
condition causes jaundice and liver cell damage, and is a medical emergency, requiring the
endoscopic retrograde cholangiopancreatography (ERCP) procedure or surgical treatment. This
obstruction may lead to jaundice, elevation in alkaline phosphatase, increase in conjugated
bilirubin in the blood and increase in cholesterol in the blood. It can also cause acute
pancreatitis and ascending cholangitis. The common bile duct is normal in size so a stone is
unlikely.

Adenocarcinoma of the head of the pancreas (choice C) is sometimes called a "silent killer"
because early pancreatic cancer often does not cause symptoms, and the later symptoms are
usually nonspecific and varied. Therefore, pancreatic cancer is often not diagnosed until it is
advanced. Common symptoms include pain in the upper abdomen that typically radiates to the
back (seen in carcinoma of the body or tail of the pancreas), loss of appetite and/or nausea
and vomiting, significant weight loss, painless jaundice (yellow skin/eyes, dark urine) when a
cancer of the head of the pancreas (about 60% of cases) obstructs the common bile duct as it
runs through the pancreas. This may also cause pale-colored stool and steatorrhea.

 Trousseau sign, in which blood clots form spontaneously in the portal blood vessels,
the deep veins of the extremities, or the superficial veins anywhere on the body, is
sometimes associated with pancreatic cancer.

 Diabetes mellitus, or elevated blood sugar levels. Many patients with pancreatic cancer
develop diabetes months to even years before they are diagnosed with pancreatic
cancer, suggesting new onset diabetes in an elderly individual may be an early
warning sign of pancreatic cancer.

Liver function tests can show a combination of results indicative of bile duct obstruction (raised
conjugated bilirubin, γ-glutamyl transpeptidase and alkaline phosphatase levels). CA19-9
(carbohydrate antigen 19.9) is a tumor marker that is frequently elevated in pancreatic
cancer. Elevated liver enzymes, however, are not often elevated in pancreatic cancer confined
to the head of the pancreas.

Cholestasis (choice D) is a condition where bile cannot flow from the liver to the duodenum.
The two basic distinctions are an obstructive type of cholestasis where there is a mechanical
blockage in the duct system such as can occur from a gallstone or malignancy and metabolic
types of cholestasis which are disturbances in bile formation that can occur because of genetic
defects or acquired as a side effect of many medications.

Acute cholestasis can be suspected when there is an elevation of both 5'-nucleotidase and ALP
enzymes. With a few exceptions, the optimal test for cholestasis would be elevations of serum
bile acid levels. The gamma-glutamyl transferase(GGT) enzyme was previously thought to be
helpful in confirming a hepatic source of ALP; however, GGT elevations are markedly sensitive
and lack the necessary specificity to be a useful confirmatory test for ALP. Normally GGT and
ALP are anchored to membranes of hepatocytes and are released in small amounts in
hepatocellular damage. In cholestasis, synthesis of these enzymes is induced and they are
made soluble. GGT is elevated because it leaks out from the bile duct cells due to pressure
from inside bile duct.

In a later stage of cholestasis AST, ALT and bilirubin may be elevated due to liver damage as a
secondary effect of cholestasis but not in the acute phase.
QUESTION A 63-year-old alcoholic man presents to the Emergency
3 of 45
Questions
Department with increasing abdominal girth and right upper
Notes quadrant pain. On physical exam, the patient has a narrow
pulse pressure and an S3 and S4 are heard on ascultation of
the heart. Examination of the right neck veins reveals venous
pulsations that are best seen when the head of the bed is
elevated to ninety degrees. The abdomen is distended with
shifting dullness and riddled with spider veins.

The synthetic function of the liver is measured by which of the


following parameters according to the Child’s classification?
ascites levels

elevated serum transaminases (ALT, AST)

bilirubin diglucuronide levels

serum albumin and prothrombin time

Hepatic encephalopathy (ammonia levels)

Explanation

The correct answer is choice D

Cirrhosis is a consequence of chronic liver disease characterized by replacement of liver tissue


by fibrosis, scar tissue and regenerative nodules,leading to loss of liver function. Cirrhosis is
most commonly caused by alcoholism, hepatitis B and C, and fatty liver disease but has many
other possible causes. Some cases are idiopathic. Cirrhosis is often preceded by hepatitis and
fatty liver (steatosis), independent of the cause. If the cause is removed at this stage, the
changes are still fully reversible.

The pathological hallmark of cirrhosis is the development of scar tissue that replaces normal
parenchyma, blocking the portal flow of blood through the organ and disturbing normal
function. Recent research shows the pivotal role of the stellate cell, a cell type that normally
stores vitamin A, in the development of cirrhosis. Damage to the hepatic parenchyma leads to
activation of the stellate cell, which becomes contractile (called myofibroblast) and obstructs
blood flow in the circulation. In addition, it secretes TGF-β1, which leads to a fibrotic response
and proliferation of connective tissue. Furthermore, it disturbs the balance between matrix
metalloproteinases and the naturally occurring inhibitors, leading to matrix breakdown and
replacement by connective tissue-secreted matrix. The fibrous tissue bands (septa) separate
hepatocyte nodules, which eventually replace the entire liver architecture, leading to
decreased blood flow throughout. The spleen becomes congested, which leads to
hypersplenism and increased sequestration of platelets. Portal hypertension is responsible for
most severe complications of cirrhosis.

The Child-Pugh score is used to assess the prognosis of chronic liver disease, mainly cirrhosis.
Although it was originally used to predict mortality during surgery, it is now used to determine
the prognosis, as well as the required strength of treatment and the necessity of liver
transplantation. Hepatic decompensation and failure may be predicted by identifying patients
with jaundice and marginal liver hepatic synthetic function.

The score employs five clinical measures of liver disease:

 Bilirubin (total) <34

 Serum albumin >35

 INR <1.7

 Ascites

 Hepatic encephalopathy

The liver plays a vital role in synthesis of proteins (e.g., albumin, clotting factors and
complement), detoxification and storage (e.g., vitamin A). In addition, it participates in the
metabolism of lipids and carbohydrates. Therefore, a decrease in albumin and clotting factors
signify a a decreased synthetic liver function in the Child’s scheme.

Ascites (choice A) is an accumulation of fluid in the peritoneal cavity. Ascitic fluid can
accumulate as a transudate or an exudate. Amounts of up to 25 liters are possible. Roughly,
transudates are a result of increased pressure in the portal vein (>8 mmHg, usually around 20
mmHg), e.g. due to cirrhosis, while exudates are actively secreted fluid due to inflammation or
malignancy. The presence of ascites provides no information on synthetic liver function status.

Elevated serum transaminases (choice B) , commonly the transaminases alanine transaminase


(ALT) and aspartate transaminase (AST), may be an indicator of liver damage. The liver has a
variety of transaminases to synthesize and break down amino acids and to interconvert energy
storage molecules. The concentrations of these in the serum are normally low. However, if the
liver is damaged, the hepatocyte cell membrane becomes more permeable and some of the
enzymes leak out into the blood stream. Elevated serum transaminases provide no information
on synthetic liver function status.

Bilirubin diglucuronide (choice C) is a conjugated form of bilirubin. In the liver it is conjugated


with glucuronic acid by the enzyme Glucuronyltransferase, making it soluble in water. Much of
it goes into the bile and thus out into the small intestine. Some of the conjugated bilirubin
remains in the large intestine and is metabolised by colonic bacteria to urobilinogen, which is
further metabolized to stercobilinogen, and finally oxidised to stercobilin. This stercobilin gives
feces its brown color. Some of the urobilinogen is reabsorbed and excreted in the urine along
with an oxidized form, urobili. Bilirubin diglucuronide provides no information on synthetic liver
function status.

Hepatic encephalopathy (choice E) is the occurrence of confusion, altered level of


consciousness and coma as a result of liver failure. It is caused by accumulation of ammonia in
the bloodstream that are normally removed by the liver. The diagnosis of hepatic
encephalopathy requires the presence of impaired liver function and the exclusion of an
alternative explanation for the symptoms. Blood tests (ammonia levels) may assist in the
diagnosis. Attacks are often precipitated by an intercurrent problem, such as infection or
constipation. Hepatic encephalopathy provides no information on synthetic liver function
status.
QUESTION On postoperative day three for a coronary artery bypass, a 56
4 of 45
Questions
year old male develops a pounding heart and shortness of
Notes breath. He feels that he is going to pass out but denies having
chest pains. His BP is 120/80 with a pulse of 210. An EKG is
performed as shown below. Attempts are made to lower his
tachycardia with carotid neck massage and splashing cold
water on his face, but the attempts fail.

What is the next appropriate step in management?


Metoprolol

Verapamil

Cardioconversion

Adenosine

Amidoarone

Explanation

The correct answer is choice D

Tachycardias may be classified as either narrow complex tachycardias (supraventricular


tachycardias) or wide complex tachycardias. "Narrow" and "wide" refer to the width of the QRS
complex on the ECG. Narrow complex tachycardias tend to originate in the atria, while wide
complex tachycardias tend to originate in the ventricles. Tachycardias can be further classified
as either regular or irregular. Supraventricular tachycardia (SVT), which is a tachycardia paced
from the atria or the AV node. SVT rhythms include: Atrial fibrillation, AV nodal reentrant
tachycardia, AV reentrant tachycardia, Junctional tachycardia.

Symptoms can come on suddenly and may go away without treatment. They are caused for a
reason other than stress, exercise, or emotion. They can last a few minutes or as long as 1 or
2 days, sometimes continuing until treated. The rapid beating of the heart during SVT can
make the heart a less-effective pump, decreasing cardiac output and blood pressure. The
following symptoms are typical with a rapid pulse of 150–251 or more beats per minute:
pounding heart, shortness of breath, chest pain, rapid breathing, dizziness, and loss of
conciousness.

In general, SVT is not life threatening, but episodes should be treated or prevented. The SVTs
can be separated into two groups, based on whether they involve the AV node for impulse
maintenance or not. Those that involve the AV node can be terminated by slowing conduction
through the AV node. Those that do not involve the AV node will not usually be stopped by AV
nodal blocking maneuvers. These maneuvers are still useful however, as transient AV block
will often unmask the underlying rhythm abnormality.

In the case of narrow complex tachycardias (junctional, atrial or paroxysmal), the treatment in
general is to first give the patient adenosine (to slow conduction through the AV node) and
then perform Valsalva maneuvers to slow the rhythm. If this does not convert the patient,
amiodarone, calcium channel blockers or beta-blockers are commonly employed to stabilize
the patient. Again as in atrial fibrillation, if a patient is unstable, the decision to electrically
cardiovert him/her should be made.

Metoprolol (choice A) iis a selective β1 receptor blocker that is used for rate control in
supraventricular tachycardia. Metoprolol is used for followup therapy after adenosine has
aborted the supraventricular tachycardia.

Verapamil (choice B) is an L-type calcium channel blocker. Since calcium channels are
especially concentrated in the sinoatrial and atrio-ventricular nodes, these agents can be used
to decrease impulse conduction through the AV node, thus protecting the ventricles from atrial
tachyarrhythmias. Verapamil is used for followup therapy after adenosine has aborted the
supraventricular tachycardia.

cardioconversion (choice C) is a medical procedure by which an abnormally fast heart rate or


cardiac arrhythmia is converted to a normal rhythm, using electricity or drugs. Synchronized
electrical cardioversion uses a therapeutic dose of electric current to the heart, at a specific
moment in the cardiac cycle. Pharmacologic cardioversion, also called chemical cardioversion,
uses antiarrhythmia medication instead of an electrical shock. If the patient is stable,
adenosine may be administered first, as the medicine performs a sort of "chemical
cardioversion" and may stabilize the heart and let it resume normal function on its own
without using electricity. Synchronized electrical cardioversion is used to treat
hemodynamically significant supraventricular (or narrow complex) tachycardias,

amidoarone (choice E) is an antiarrhythmic agentused for various types of tachyarrhythmias,


both ventricular and supraventricular (atrial) arrhythmias that do not involve the AV node.
Amiodarone is categorized as a class III antiarrhythmic agent, and prolongs phase 3 of the
cardiac action potential. Amiodarone shows beta blocker-like and potassium channel blocker-
like actions on the SA and AV nodes, increases the refractory period via sodium- and
potassium-channel effects, and slows intra-cardiac conduction of the cardiac action potential,
via sodium-channel effects.
QUESTION A 30 year old man with bright red blood per rectum (BRBPR)
5 of 45
Questions
with bowel movements presents to the emergency room after
Notes experiencing an episode of syncope. He has no significant
medical and his family history is noncontributory. He also
complains of some constipation but generally no change in
bowel movements, weight loss, or abdominal pain. He denies
black tarry stools or hematochezia but his stools are guaiac
positive. His vitals signs are stable and the rest of the exam is
normal.

What is the next appropriate step in management after IV


access is established?

Colonoscopy

Colonoscopy and banding of hemorrhoids

Sigmoidoscopy and banding of hemorrhoids

Reassurance

Surgical intervention

Explanation

The correct answer is choice C

Lower gastrointestinal bleedin (LGIB), refers to any form of bleeding in the lower
gastrointestinal tract. LGIB is a common ailment seen at emergency departments.
Approximately 85% of Lower gastrointestinal bleeding involves the colon, 10% are from bleeds
that are actually Upper gastrointestinal bleeds, and 3 - 5% involve the small intestines. A
lower Gastrointestinal Bleed is referred as any bleed that occures distal to the ligament of
Treitz and superior to the anus. This includes the last 1/4 of the duodenum and the entire area
of the jejunum, ileum, colon, rectum, and anus.
The stool of a person with a lower gastrointestinal bleed is a good not infallible indication of
where the bleeding is occurring. Black tarry appearing stools medically referred to as melena
usually indicates blood that has been in the GI tract for at least 8 hours. Melena is four-times
more likely to come from an upper gastrointestinal bleed than from the lower GI tract;
however, it can also occur in either the duodenum and jejunum, and occasionally the portions
of the small intestine and proximal colon. Bright red stool, called hematochezia, is the sign of a
fast moving active GI bleed. The bright red or maroon color is due to the short time taken
from the site of the bleed and the exiting at the anus. The presence of hematochezia is six-
times greater in a LGIB than with a UGIB,

Occasionally, a person with a LGIB will not present with any signs of internal bleeding. In
these cases, a diagnostic assessment or pre-assessment should watch for other signs and
symptoms that the patient may present with. These include, but are not limited to,
hypotension, tachycardia, angina, syncope, weakness, confusion, stroke, myocardial infarction,
shock, and heart attack.

Evaluations will most often be conducted by either a clinic triage nurse, emergency
department nurse, and/or a physician or other clinican. The initial assessment will include the
appearance of the individual, their vital signs, and mental status. A patient history will help
reveal a disposition or history of LGIBs or potential differential diagnosis.

Orthostatic vital signs are often used as a indicator of hypovolemia. Laboratory test will also
help give indications of a LGIB. Hemoglobin, hematocrit, and platelets are very good physical
signs of hypovolemia or blood loss amenia. Aspiration of the stomach contents by way of a
nasogastric tube (NG tube) will help differentiate between either a UGIB and a LGIB. A
negative presence of blood will help to rule out an UGIB. IIf a patient is suspected of having
severe blood loss they will most likely be placed on a vital sign monitor and administered
oxygen either by nasal cannula or simple face mask. An intravenous catheter will be placed
into an easily accessible area and IV fluids will be administered to replace lost volume.

Although the differential diagnosis for LGIB is extensive, this patient most likely has bleeding
hemorrhoids that can be visualized by sigmoidoscopy and treated by banding. A
sigmoidoscopy only examines up to the sigmoid, the most distal part of the colon, while
colonoscopy examines the whole large bowel. Hemorrhoids rarely extend past the distal colon
so a colonscopy is unnecessary.

Full colonoscopy (choice A) is not necessary for symptomatic bleeding hemorrhoids.


Colonscopies are not completely benign procedures and carry the risk of gastrointestinal
perforation. The risk of perforation roughly doubles with colonscopies compared to
sigmoidoscopies. It is also inappropriate to not treat a symptomatic bleeding hemorrhoid.
Full colonoscopy and banding of hemorrhoids if present (choice B) is not appropriate
because the benefit of visualizing the colon past the anorectal junction is not outweighed by
the risk of perforation for bleeding hemorrhoids.

Reassurance (choice D) is inappropriate for symptomatic bleeding hemorrhoids. There are a


number of therapies that can be undertaken to stop bleeding hemorrhoids including rubber
band ligation and scleropterapy. Rubber band ligation is a procedure in which elastic bands are
applied onto an internal hemorrhoid at least 1 cm above the dentate line to cut off its blood
supply. Within 5–7 days, the withered hemorrhoid falls off. The cure rate has been found to be
about 87%. Sclerotherapy involves the injection of a sclerosing agent (such as phenol) into the
hemorrhoid. This causes the vein walls to collapse and the hemorrhoids to shrivel up. The
success rate at four years is 70%.

Surgical intervention (choice E) for bleeding hemorrhoids is not warranted for a stable patient.
If a patient is suspected of having severe blood loss they will most likely be placed on a vital
sign monitor and administered oxygen either by nasal cannula or simple face mask. An
intravenous catheter will be placed into an easily accessible area and IV fluids will be
administered to replace lost volume. Surgical intervention is warranted in some cases. It is
most likely that a sugical consult will be ordered if the patient is unable to be stabilized by
non-invasive techniques, or a perforation is found that requires surgery.
QUESTION A 62 year old healthy white male presents with a chief
6 of 45
Questions
complaint of difficulty reading for the past two weeks. He only
Notes wears spectacle correction for near tasks. He went to Walmart
OTC and obtained a new pair of glasses w/o relief of his
symptoms. The onset was abrupt, painless and he denies
headache, or other neurologic symptoms. He notes some
nausea since the occurrence. Medical history is significant for
mild hypertension and he is using lisinopril daily.

Relevant vital signs:

 BP 140/100 mm hg
 Glucose serum level 225 mg% (50-100)
 Visual acuity OD 20/25
 Visual acuity OS 20/30
 IOP OD 16
 IOP OS 16 mm hg (Goldman tonometry)
The patient’s appearance is shown in the accompanying
photograph.

Which of the following is the correct answer:

Sixth Nerve Palsy

Fourth Nerve Palsy OS

Fourth Nerve Palsy OD

Third Nerve Palsy OD


Click on image to Zoom
Medial longitudinal fascicle
(MLF) Syndrome

Explanation

The correct answer is choice C

The patient noted diploplia when reading because the superior oblique muscle is most active in
convergence and infragaze…the reading position. He presents with a left head tilt to minimize
the effect of a superior oblique palsy in the right eye. In this case, decreased in torsion of the
RSO causes the patient to tilt his head in the direction of the muscles action, thereby
minimizing or eliminating the symptom.

Typical etiologies are most often traumatic, vascular, tumors rarely, and are often idiopathic
with a negative workup.

Treatment consists of weakening the agonist muscle, the inferior oblique by recessing it.
Surgery for superior oblique palsy is often only partially successful and the patient must be
fitted with spectacle prisms to eliminate the diploplia.

Choice A is incorrect because the 6th nerve (abducens) controls lateral abduction and would
create a head turn, not a head tilt. The affected eye would turn in (esotropia) in the primary
position, causing horizontal diploplia.

Choice B is incorrect because the head tilt would be in the opposite direction, (right head tilt)
and the right eye would be deviated superiorly.

Choice D is unlikely because it would affect all muscles except the superior oblique and lateral
rectus muscles. There would also be a ptosis since the levator palpebra muscle is also
innervated by the 3rd nerve (oculomotor).

Medial longitudinal fascicle (MLF) forms the ventral border of the occulomotor and tochlear
nucleus in the pons. MLF serves an important function in coordinating the movement of the
eye in vision by interconnecting the occulomotor nuclei, trochlear nuclei and abducens nuclei.
Bilateral (unilateral much less common) internuclear ophthalmoplegia, so called MLF syndrome
(choice E), is a classic, and often the initial, manifestation of multiple sclerosis. It is caused by
demyelination in MLF. Manifestations of MLF syndrome are featured by weakness in adduction
of the ipsilateral eye with nystagmus on abduction of the contralateral eye, and incomplete or
slow abduction of the ipsilateral eye upon lateral gaze.
QUESTION A 3 year old child presents to the emergency room with a 2 day
7 of 45
Questions
history of painless rectal bleeding.
Notes
Massive painless rectal bleeding is most likely to be seen in
children with which of the following conditions:

duodenal atresia/intussusception

pyloric stenosis/midgut volvulus

lactose intolerance/annular pancreas


Prune-Belly (Eagle-Barrett) syndrome

Meckel's diverticulum/intestinal duplication

Explanation

The correct answer is choice E.

Meckel’s diverticulum is the most common congenital abnormality of the small intestine; it is
caused by an incomplete obliteration of the vitelline duct. In most cases, it contains ectopic
gastric tissue that secretes gastric acid leading to ulceration of the surrounding tissues,
Bleeding from these ulcer usually ends into painless bleeding per rectum.

Alimentary tract duplication is rare developmental error that has been assigned several
different names, including enterocystomas, enterogenous cysts, supernumerary accessory
organs, ileum duplex, giant diverticula, and unusual Meckel’s diverticulum. In this condition
abrupt hemorrhage can be encountered in the case of a cyst lined with gastric mucosa that
ulcerates and eventually erodes into adjacent organs and/or vessels.

Answer A is false: In duodenal atresia there is no bleeding per rectum and even there is no
passage of meconium. In intussusception there is painful colicky passage of stools that look
like currant jelly. This is a mixture of mucus, sloughed mucosa, and shed blood.

Answer B is false: In pyloric stenosis there is nonbilious vomiting. There may be Slight
hematemesis of either bright red flecks or a coffee-ground appearance but no rectal bleeding
occurs.

Midgut vovulus is presented by abdominal distension, visible peristaltic in the epigastrium,


bilious or nonbilious emesis and bloody diarrhea associated with severe abdominal pain.

Answer C is false: Annular pancreas is similar to duodenal atresia and not presented by
bleeding per rectum and even there is no passage of meconium. Lactose intolerance is
presented by bloating, abdominal discomfort, meteorism, flatulence and passage of loose,
watery and acidic stool that occur from 1 hour to a few hours after ingestion of milk or dairy
products associated with functional recurrent abdominal pain.

Answer D is false: Prune belly syndrome typically occurs in boys with a thin or lax abdominal
wall. It is characterized by long prostatic urethra, prostatic hypoplasia, tortuous and dilated
ureters, hydronephrosis and varying degrees of renal dysplasia. It may be associated wit GI
anomalies as malrotation with mesenteric defect, imperforate anus, gastroschisis,
Hirschsprung disease, and constipation.
QUESTION A 3 week old boy has had projectile vomiting and weight loss
8 of 45
Questions
over the past week. You suspect pyloric stenosis. Which of the
Notes following is/are correct regarding pyloric stenosis?

1. a mixed respiratory and metabolic alkalosis is the usual acid


base disorder associated with this condition
2. even if a pyloic tumor is palpable, roentgenographic studies
of the upper gastrointestinal tract should be obtained prior to
surgery
3. a pyloric tumor is usually palpable in the left upper quadrant
4. ultrasonography is a reliable diagnostic procedure

1,2,3 only

1,3 only

2,4 only

4 only

All of the above are correct

Click on image to Zoom

Explanation
The correct answer is choice D.

Pyloric stenosis involves hypertrophy of the circular muscle of the


pylorus, resulting in narrowing and obstruction of the pyloric channel
by compression of longitudinal folds of mucosa. Gastric outlet
obstruction results in emesis, which is characteristically nonbilious and
projectile. Protracted emesis, as well as failure of the stomach to
empty into the duodenum, results in progressive dehydration,
electrolyte abnormalities, acid-base disorders, weight loss, and,
potentially, shock.

Phrase 4 only is true as the ultrasonography can show the following


criteria which are reliable and specific for pyloric stenosis :

 Muscle thickness (serosa to mucosa) greater than 3 mm.

 Target sign on transverse images of the pylorus.(See the


picture)

 Pyloric channel length greater than 17 mm.

 Pyloric thickness (serosa to serosa) of 15 mm or greater.

 Failure of the channel to open during a minimum of 15 minutes


of scanning.

 Retrograde or hyperperistaltic contractions.

 Antral nipple sign (a prolapse of redundant mucosa into the


antrum, which creates a pseudomass)
Phrase 1 is false as there is only metabolic alkalosis in such case.
The alkalosis is caused by loss H and Cl ions due to repeated vomiting.
There is no respiratory element in the development of the alkalosis.

Phrase 2 is false as once a clear history is obtained and the pyloric


tumor is palpated, the diagnosis is established and there would be no
need for further imaging studies.

Phrase 3 is false as the pyloric tumor is located in the epigastrium


just above the umbilicus, either in the midline or just to the right.

QUESTION A 62-year-old woman notices blood in the toilet bowel after a


9 of 45
Questions
bowel movement. She undergoes a colonoscopy and is found to
Notes have a rectal mass in the sigmoid colon. Biopsies of the lesion
are taken and reveal adenocarcinoma.

Which of the following modalities would best assess the local


extent of disease in this patient?

CT scan of the abdomen and pelvis

Endoscopic ultrasound (EUS)

MRI scan of the abdomen and pelvis

Positron emission tomography (PET) scan of the abdomen


and pelvis

Double contrast barium enema


Explanation

The correct answer is choice B

Colorectal cancer, also called colon cancer or large bowel cancer, includes cancerous growths
in the colon, rectum and appendix. It is the fourth most common form of cancer in the United
States and the third leading cause of cancer-related death in the Western world. Colorectal
cancers arise from adenomatous polyps in the colon. These mushroom-shaped growths are
usually benign, but some develop into cancer over time. Localized colon cancer is usually
diagnosed through colonoscopy.

Invasive cancers that are confined within the wall of the colon (TNM stages I and II) are
curable with surgery. If untreated, they spread to regional lymph nodes (stage III), where up
to 73% are curable by surgery and chemotherapy. Cancer that metastasizes to distant sites
(stage IV) is usually not curable, although chemotherapy can extend survival, and in rare
cases, surgery and chemotherapy together have seen patients through to a cure. Radiation is
used with rectal cancer.

Colon cancer staging is an estimate of the amount of penetration of a particular cancer. It is


performed for diagnostic and research purposes, and to determine the best method of
treatment. The systems for staging colorectal cancers depend on the extent of local invasion,
the degree of lymph node involvement and whether there is distant metastasis.

Definitive staging can only be done after surgery has been performed and pathology reports
reviewed. An exception to this principle would be after a colonoscopic polypectomy of a
malignant pedunculated polyp with minimal invasion. Preoperative staging of rectal cancers
may be done with endoscopic ultrasound. Endoscopic ultrasound is a medical procedure in
which endoscopy is combined with ultrasound to obtain images of the internal organs in the
chest and abdomen. It can be used to visualize the wall of these organs, or to look at adjacent
structures. Combined with Doppler imaging, nearby blood vessels can also be evaluated.
Adjunct staging of metastasis include Abdominal Ultrasound, CT, PET Scanning, and other
imaging studies.

CT scan of the abdomen and pelvis (choice A) can be used for adjunct staging. Standard
computed axial tomography is an x-ray method that can be used to determine the degree of
spread of cancer.

MRI scan of the abdomen and pelvis (choice C) can be used for adjunct staging but it is not
the initial imaging modality to order to assess for the local extent of disease.

Positron emission tomography (PET) scan of the abdomen and pelvis (choice D) is a 3-
dimensional scanning technology where a radioactive sugar is injected into the patient, the
sugar collects in tissues with high metabolic activity, and an image is formed by measuring the
emission of radiation from the sugar. Because cancer cells often have very high metabolic
rates, this can be used to differentiate benign and malignant tumors. PET is not used for
screening and does not (yet) have a place in routine workup of colorectal cancer cases.

Double contrast barium enema (choice D) can detect a cancer or a precancerous polyp but
miss the (less common) flat polyp. An enema containing barium sulfate is administered, then
air is insufflated into the colon, distending it. The result is a thin layer of barium over the inner
lining of the colon which is visible on X-ray films.
QUESTION You are a senior resident in pediatric surgery and you get a call
10 of 45
Questions
from the neonatologist about a neonate having severe
Notes respiratory distress and cyanosis. On examination you found a
scaphoid abdomen, barrel-shaped chest and signs of
respiratory distress including retractions and grunting.
Auscultation of chest revealed diminished air entry on the left
side. An orogastric tube was introduced then x-ray film was
made. The tube was seen coiled in the left side of chest. Your
provisional diagnosis was Congenital diaphragmatic hernia.

Regarding Congenital diaphragmatic hernias, all of the


following statements are true except:

Usually occur through the


foramen of Bochdalek
Are more common on the left
than the right
Are frequently associated
with other congenital
anomalies
First diagnosis can be only Click on image to Zoom
made at birth
Neonates usually require
sedation, ventilation and
intestinal decompression prior
to surgery

Explanation

The correct answer is choice D.

Congenital diaphragmatic hernia can be diagnosed during prenatal


period using ultrasonography with high sensitivity. The peristalsis of
bowel loops can be seen in chest. In addition there may be other
suggesting criteria as polyhydramnios, intrathoracic gastric air bubble
and a mediastinal shift away from the herniation.

Choice A is a true phrase as in 85% of cases herniation occurs


through foramen of Bochdalek, a congenital defect in diaphragm in its
left postero-lateral aspect due to incomplete closure of pericardio
peritoneal canal. The remaining 15% includes hernia through foramen of Morgagni,
congenital hiatus hernia and eventration of diaphragm.

Choice B is a true phrase as foramen of Bochdalek is present on the


left side of diaphragm and herniation through it represents 85% of cases
as mentioned.

Choice C is a true phrase as there are many congenital anomalies that


may be associated with this type of hernia. 30% of cases are
associated with chromosomal anomalies as triosomy 18 or 21 or
monosomy x (Turner syndrome). 10% of cases are syndromatic as
Fryns syndrome or Cornelia de Lang syndrome. 25% of cases are
associated with cardiac anomalies as atrial septal defect or
transposition of great vessels. Also there are many neural, urological
or pulmonary defects that may be present.

Choice E is a true phrase as once the case is diagnosed, an orogastric


tube should be introduced to deflate stomach and bowel loops allowing
lung expansion followed by mechanical ventilation. Sedation is also
required to decrease swallowing of air and allow adequate mechanical
ventilation.
QUESTION A 76 year old man is brought to the ER from a nursing home for
11 of 45
Questions
increasing abdominal pain and distension. He has been
Notes constipated and has not had a bowel movement in over a week.
Physical exam reveals a soft, distended tympanic abdomen
with severe tenderness in all four quadrants. Labs are
significant for [HCO3-] of 21 mmol/l and blood pH of 7.31. A CAT
scan of the abodmen shows a sigmoid volvulus.

What is the next appropriate step in management?

sigmoidoscopy

endoscopic reduction

fecal disimpaction

barium enema

emergent laparotomy

Explanation

The correct answer is choice E


A volvulus is a bowel obstruction in which a loop of bowel has abnormally twisted on itself.
Midgut volvulus occurs in patients (usually in infants) that are predisposed because of
congenital intestinal malrotation. Segmental volvulus occurs in patients of any age, usually
with a predisposition because of abnormal intestinal contents (e.g. meconium ileus) or
adhesions. Volvulus of the cecum, transverse colon, or sigmoid colon occurs, usually in adults,
with only minor predisposing factors such as redundant (excess, inadequately supported)
intestinal tissue and constipation.

Volvulus occurs most frequently in middle-aged and elderly men. Volvulus can also arise as a
rare complication in persons with redundant colon, a normal anatomic variation resulting in
extra colonic loops. Sigmoid volvulus is the most-common form of volvulus of the
gastrointestinal tract and is responsible for 8% of all intestinal obstructions. Sigmoid volvulus
is particularly common in elderly persons and constipated patients. Patients experience
abdominal pain, distension, and absolute constipation.

Regardless of cause, volvulus causes symptoms by two mechanisms.

 One is bowel obstruction, manifested as abdominal distension and vomiting.

 The other is ischemia (loss of blood flow) to the affected portion of intestine.

Volvulus causes severe pain and progressive injury to the intestinal wall, with accumulation of
gas and fluid in the portion of the bowel obstructed. Ultimately, this can result in necrosis of
the affected intestinal wall, acidosis, and death. Acute volvulus therefore requires immediate
surgical intervention to untwist the affected segment of bowel and possibly resect any
unsalvageable portion.

Emergent surgical intervention is indicated for acute gastric volvulus and is still considered a
surgical emergency by many surgeons. With chronic gastric volvulus, surgery is performed to
prevent complications. The principles associated with the treatment of gastric volvulus include
decompression, reduction, and prevention of recurrence, which is best accomplished with
surgical therapy.

sigmoidoscopy (choice A) is a minimally invasive medical examination of the large intestine


from the rectum through the last part of the colon through the use of an endoscopic camera.
Sigmoidoscopy enables the physician to look at the inside of the large intestine from the
rectum through the last part of the colon, called the sigmoid to find the cause of diarrhea,
abdominal pain, or constipation. However, it is not necessary after the diagnosis of volvus has
been established by CT or plain film abdominal xrays.

endoscopic reduction (choice B) can be attempted in selected patients but is not appropriate in
in patients who appear clinically ill or are found to have vascular compromise during
endoscopy. Such treatment can be accomplished by advancing the scope beyond the point of
torsion and then rotating it to untwist the stomach. Nonoperative treatment might be
appropriate for older patients that have intestinal malrotation and are truly asymptomatic.
Definitive treatment for sigmoid volvulus remains surgical with resection and primary
anastomosis.

fecal disimpaction (choice C) involves removing a solid, immobile bulk of human feces that can
develop in the rectum as a result of chronic constipation through the use of laxatives or
manual disimpaction. While it may relieve symptomatic relief of constipation, it will not resolve
the volvulus.

barium enema (choice D) is used to examine and diagnose problems with the human colon
(large intestine). X-ray pictures are taken while barium sulfate fills the colon via the rectum.
Sigmoidoscopy, rather than barium enema examination, is the procedure of choice if an
ileosigmoid knot is suspected. Both are unnecessary when diagnosis of volvulus has been
established by CT or abdominal radiograph.
QUESTION An asymptomatic 66 year-old man has a routine complete blood
12 of 45
Questions
count which reveals a leukocytosis of 50,000 white blood cells
Notes per microliter. On fluorescent in situ hybridization assay, he is
found to have the Philadelphia chromosome in peripheral blood
mononuclear cells, and he is diagnosed with chronic
myelogenous leukemia. He subsequently undergoes radiation
therapy and a bone marrow transplantation. Within thirty days,
he develops a maculopapular rash, abdominal pain, and severe
nausea and vomiting.

Which one of the following is the cause of the patient’s


symptoms?
Complement

Preformed antibodies

Cytokines

NK cells

Macrophages

Explanation

The correct answer is choice C

Graft-versus-host disease (GVHD) is a common complication of allogeneic bone marrow


transplantation in which functional immune cells in the transplanted marrow recognize the
recipient as "foreign" and mount an immunologic attack. GVHD is an inflammatory disease
that is unique to allogeneic transplantation. This can occur even if the donor and recipient are
HLA-identical because the immune system can still recognize other differences between their
tissues. It is aptly named graft-versus-host disease because bone marrow transplantation is
the only transplant procedure in which the transplanted cells must accept the body rather than
the body accepting the new cells.

Clinically, graft-versus-host-disease is divided into acute and chronic forms.

 The acute or fulminant form of the disease (aGVHD) is normally observed within the
first 100 days post-transplant and is a major challenge to transplants owing to
associated morbidity and mortality. Acute graft-versus-host disease typically occurs in
the first 3 months after transplantation and may involve the skin, intestine, or the
liver, and is often fatal.

 The chronic form of graft-versus-host-disease (cGVHD) normally occurs after 100


days. The appearance of moderate to severe cases of cGVHD adversely influences
long-term survival.
Classically, acute graft-versus-host-disease is characterized by selective damage to the liver,
skin and mucosa, and the gastrointestinal tract. Chronic graft-versus-host-disease also attacks
the above organs, but over its long-term course can also cause damage to the connective
tissue and exocrine glands. Acute GVHD of the GI tract can result in severe intestinal
inflammation, sloughing of the mucosal membrane, severe diarrhea, abdominal pain, nausea,
and vomiting. This is typically diagnosed via intestinal biopsy. Liver GVHD is measured by the
bilirubin level in acute patients. Skin GVHD results in a diffuse maculopapular rash, sometimes
in a lacy pattern.

After bone marrow transplantation, T cells present in the graft, either as contaminants or
intentionally introduced into the host, attack the tissues of the transplant recipient after
perceiving host tissues as antigenically foreign. The T cells produce an excess of cytokines,
including TNF-α and interferon-gamma (IFNγ). A wide range of host antigens can initiate graft-
versus-host-disease, among them the human leukocyte antigens (HLAs). However, graft-
versus-host disease can occur even when HLA-identical siblings are the donors. HLA-identical
siblings or HLA-identical unrelated donors often have genetically different proteins (called
minor histocompatibility antigens) that can be presented by MHC molecules to the recipient's
T-cells, which see these antigens as foreign and so mount an immune response.

High-dose corticosteroids such as prednisone are a standard treatment; however this immuno-
suppressive treatment often leads to deadly infections. Chronic graft-versus-host disease may
also develop after allogeneic transplant. It is the major source of late treatment-related
complications, although it less often results in death. In addition to inflammation, chronic
graft-versus-host disease may lead to the development of fibrosis, or scar tissue, similar to
scleroderma; it may cause functional disability and require prolonged immunosuppressive
therapy.

Complement (choice A) consists of a number of small proteins found in the blood, generally
synthesized by the liver, and normally circulating as inactive precursors. When stimulated by
one of several triggers, proteases in the system cleave specific proteins to release cytokines
and initiate an amplifying cascade of further cleavages. The end-result of this activation
cascade is massive amplification of the response and activation of the cell-killing membrane
attack complex. Over 25 proteins and protein fragments make up the complement system,
including serum proteins, serosal proteins, and cell membrane receptors. They account for
about 5% of the globulin fraction of blood serum. Graft-versus-host-disease is not mediated by
the complement system

Preformed antibodies (choice B) causes hyperacute rejection in transplanted tissues. It is a


complement-mediated response in recipients with pre-existing antibodies to the donor (for
example, ABO blood type antibodies). Preformed antibodies does not cause Graft-versus-host-
disease.
NK cells (choice D) are a type of cytotoxic lymphocyte that constitute a major component of
the innate immune system. NK cells play a major role in the rejection of tumors and cells
infected by viruses. They kill cells by releasing small cytoplasmic granules of proteins called
perforin and granzyme that cause the target cell to die by apoptosis. NK cells are not involved
in Graft-versus-host-disease.

Macrophages (choice E) are phagocytes, acting in both non-specific defense (innate immunity)
as well as to help initiate specific defense mechanisms (adaptive immunity). Their role is to
phagocytose (engulf and then digest) cellular debris and pathogens either as stationary or as
mobile cells, and to stimulate lymphocytes and other immune cells to respond to the
pathogen. Macrophages are not involved in Graft-versus-host-disease.
QUESTION A 6-year-old child has fever, generalized abdominal pain and
13 of 45
Questions
tenderness, vomiting, and loose, guaiac-positive stools.
Notes Laboratory evaluation reveals a hematocrit of 29%, leukocyte
count of 17.0 x 103/mm3, and a platelet count of 62 × 103/mm3.

Which of the following is the most likely diagnosis for this


patient:

Appendicitis

Gastroenteritis

Hemolytic uremic syndrome

Intussusception

Henoch Schonlein Purpura

Explanation
The correct answer is choice C.

Hemolytic uremic syndrome, diarrhea-associated type in this case, is a


syndrome characterized by a triad of hemolytic anemia,
thrombocytopenia and acute renal failure.It is recognized as the most common
cause of acute renal failure in the pediatric population. Hemolytic-uremic syndrome is
classified as diarrhea-associated (D+ hemolytic-uremic syndrome) and non–diarrhea-
associated (D- or atypical hemolytic-uremic syndrome).

It is caused by gastrointestinal infection by shiga toxin-producing


Escherichia coli. The toxin produces hemorragic colitis causing acute
abdomen with occult blood in stools; guaiac-positive stools.

Then the toxin is absorbed to circulation and reaches the kidney


causing endothelial cell injury of the glomerular capillaries with micro
thrombi formation inside these capillaries. This leads to diminished
glomerular filtration with hemolysis of the RBCs as they pass through
the capillaries. In addition, there is increased release of a platelet
aggregating factor; thrombaxane A2 leading to consumptive
thrombocytopenia.

Answer A is false as appendicitis is not associated with


thrombocytopenia, anemia or occult blood in stools.

Answer B is false as gastroenteritis cannot explain the existing


thrombocytopenia and the high grade of anemia.

Answer D is false as intussusception is also not associated with


thrombocytopenia or leukocytosis.

Answer E is false. Although Henoch Schonlein Purpura is characterized by thrombocytopenia,


abdominal pain and occult blood in stools, yet the pupura of skin, the most prominent physical
finding in this disease, is missing in this case. Also it is not associated with evident anemia.
QUESTION A 24 year woman presents to bariatric clinic for presurgical
14 of 45
Questions
evaluation. She has has long standing issues with her weight
Notes and her current BMI is 45 kg/m2. Her medical history is
significant for type 2 diabetes and osteoarthritis. She has
difficulties breathing during sleep and is on a CPAP machine to
deliver continuous positive pressure for obstructive sleep
apnea.

Which of the following statements regarding surgery for morbid


obesity is correct?

Average 30-day mortality ranges from 0.1% to 1.1%

Among major comorbidities that are resolved or improved by


surgery, obstructive sleep apnea is least frequently affected
Compared with an open approach, laparoscopic gastric
bypass is associated with lower incidence of incisional
hernias and higher excess weight loss achieved after 3
years
Biliopancreatic diversion is associated with significant
postoperative morbidity and mortality and should be
reserved for patients who fail to achieve weight loss with
restrictive operations

bariatric surgery has few complications

Explanation

The correct answer is choice A

Obesity is a medical condition in which excess body fat has accumulated to the extent that it
may have an adverse effect on health, leading to reduced life expectancy and/or increased
health problems. Obesity increases the likelihood of various diseases, particularly heart
disease, type 2 diabetes, breathing difficulties during sleep, certain types of cancer, and
osteoarthritis. Body mass index (BMI), a measurement which compares weight and height,
defines people as overweight (pre-obese) when their BMI is between 25 kg/m2 and 30 kg/m2,
and obese when it is greater than 30 kg/m2. The surgical literature breaks down "class III"
obesity into further categories whose exact values are still disputed.

 Any BMI ≥ 35 or 40 is severe obesity

 A BMI of ≥ 35 or 40–44.9 or 49.9 is morbid obesity

 A BMI of ≥ 45 or 50 is super obese

Bariatric surgery ("weight loss surgery") is the use of surgical intervention in the treatment of
obesity. As every operation may have complications, surgery is only recommended for
severely obese people (BMI > 40) who have failed to lose weight following dietary modification
and pharmacological treatment. Weight loss surgery relies on various principles: the two most
common approaches are reducing the volume of the stomach (e.g. by adjustable gastric
banding and vertical banded gastroplasty), which produces an earlier sense of satiation, and
reducing the length of bowel that comes into contact with food (gastric bypass surgery), which
directly reduces absorption. Band surgery is reversible, while bowel shortening operations are
not. Some procedures can be performed laparoscopically. Complications from weight loss
surgery are frequent with average 30-day mortality ranging from 0.1% to 1.1%.

Surgery for severe obesity is associated with long-term weight loss and decreased overall
mortality. One study found a weight loss of between 14% and 25% (depending on the type of
procedure performed) at 10 years, and a 29% reduction in all cause mortality when compared
to standard weight loss measures.

Among major comorbidities that are resolved or improved by surgery, obstructive sleep apnea
is least frequently affected (choice B) is incorrect. Patients that undergo bariatric surgery often
experience symptomatic relief of obstructive sleep apnea with weight loss.

Compared with an open approach, laparoscopic gastric bypass is associated with lower
incidence of incisional hernias and higher excess weight loss achieved after 3 years (choice C)
is incorrect. Incisional hernias and weight loss rates are not significantly different between
open versus laproscopic approaches.

Biliopancreatic diversion is associated with significant postoperative morbidity and mortality


and should be reserved for patients who fail to achieve weight loss with restrictive operations
(choice D) is not correct. Biliopancreatic diversion (BPD) involves resecting a portion of the
stomach, creating a smaller stomach (however the patient can eat a free diet as there is no
restrictive component). The distal part of the small intestine is then connected to the pouch,
bypassing the duodenum and jejunum.

In around 2% of patients there is severe malabsorption and nutritional deficiency that requires
restoration of the normal absorption. The malabsorptive effect of BPD is so potent that those
who undergo the procedure must take vitamin and dietary minerals above and beyond that of
the normal population. Without these supplements, there is risk of serious deficiency diseases
such as anemia and osteoporosis.

bariatric surgery has few complications (choice E) is incorrect. Gallstones are a common
complication of the rapid weight loss following any type of bariatric surgery. Therefore, some
surgeons remove thegallbladder as a preventative measure. Others prefer to prescribe
medications to reduce the risk of post-operative gallstones.
QUESTION A 63-year-old man has had insulin dependent diabetes mellitus
15 of 45
Questions
for over two decades. The degree of control of his disease is
Notes characterized by the laboratory finding of a hemoglobin A1C of
10.1%. He has noted episodes of abdominal pain following
meals for the past few weeks. Postprandial pain, typically
starts 20-30 minutes after his last meal, that may last up to 60-
90 minutes. These episodes have worsened over the past year.
There is no history of nausea or vomitting.

On physical examination, you have a well appearing obese male


that is in no distress. Cardiovascular and respiratory exam are
normal. On abdominal examination, there are no masses and no
organomegaly, and he has no tenderness to palpation. Bowel
sound are normal in all 4 quadrants.

Which of the following pathologic findings is most likely to be


present in this man?
Chronic pancreatitis

Gastric carcinoma

Mesenteric artery occlusion

Diabetic gastroparesis

Gastric ulcer

Explanation

The correct answer is choice C

Chronic abdominal pain (CAP) in patients with diabetes may be due to chronic mesenteric
ischemia (CMI). All above options are differentials of CAP that occur after a meal.

Atherosclerosis is 2–4 times more common in patients with diabetes and affects mainly
carotid, coronary, iliac and lower limb arteries as well as the aorta. Another less common
complication is chronic mesenteric ischemia (CMI, intestinal angina), caused by atherosclerotic
obstruction of the celiac artery and its branches and results in episodic or constant intestinal
hypoperfusion.

During meals, as the demand for blood in the intestinal tract rises, the vessels are not able to
supply this increased demand, and angina-like pain results.The typical presentation is crampy
postprandial abdominal pain between 10 minutes and 3 hours after a meal and as a result,
they are afraid to eat (phagophobia). This leads to anorexia and weight loss over time. Later
changes in bowel habits occur, many report diarrhea or constipation. They may also complain
of frank blood in their stools as their bowel begins to develop necrosis, and occult testing of
stool may return positive results because of the sloughing of dead ischemic bowel.
At least two of the three main splanchnic arteries must be significantly occluded in order for
CMI to be symptomatic. Abdominal examination findings may be benign. Patients with a
history of chronic mesenteric ischemia may have physical findings consistent with a
malnourished state. Diagnostic procedure of choice is conventional angiography. Laboratory
examination should include the following:

 prothrombin time (PT)

 activated partial thromboplastin time (aPTT)

 complete blood count (CBC), which may reveal leukocytosis and/or hemoconcentration

 chemistries, which may show acidosis or increased amylase or lactate dehydrogenase


(LDH) levels

 chest radiography

 electrocardiography

In terms of treatment, revascularization of the occluded arteries is most often


recommended. Risk factors for arterial thrombosis include atherosclerosis, hypovolemia,
congestive heart failure, recent MI, advanced age, and intra-abdominal malignancy. a
significantly higher risk of acute mesenteric ischemia in the patients with IBD.

Most people with chronic pancreatitis (Choice A) experience upper abdominal pain, although
some people have no pain at all. The pain may spread to the back, feel worse when eating or
drinking, and become constant and disabling. In some cases, abdominal pain goes away as the
condition worsens, most likely because the pancreas is no longer making digestive enzymes.
Other symptoms include nausea, vomiting, weight loss, diarrhea and oily stools. People with
chronic pancreatitis often lose weight, even when their appetite and eating habits are normal.
The weight loss occurs because the body does not secrete enough pancreatic enzymes to
digest food, so nutrients are not absorbed normally. Poor digestion leads to malnutrition due
to excretion of fat in the stool.

The most worrisome diagnosis in someone who complains of postprandial pain is gastric cancer
(choice B). Stomach cancer is often asymptomatic or causes only nonspecific symptoms in its
early stages. By the time symptoms occur, the cancer has often reached an advanced stage
(see below) which is one of the main reasons for its poor prognosis.

Signs and symptoms of gastroparesis (choice D) include the following:

 heartburn

 pain in the upper abdomen

 nausea

 vomiting of undigested food—sometimes several hours after a meal

 early feeling of fullness after only a few bites of food

 weight loss due to poor absorption of nutrients or low calorie intake

 abdominal bloating

 high and low (variable) blood glucose levels

 lack of appetite

 gastroesophageal reflux

 spasms in the stomach area

Gastroparesis is the result of damage to the vagus nerve, which controls the movement of
food through the digestive system. Instead of moving through the digestive tract normally, the
food is retained in the stomach. Gastroparesis may occur in people with type 1 diabetes or
type 2 diabetes. The vagus nerve becomes damaged after years of high blood glucose,
resulting in gastroparesis. In turn, gastroparesis contributes to poor blood glucose control.
Given the clinical scenario, gastric ulcer (Choice E) is not likely to be the cause.
QUESTION On postoperative day two of an uncomplicated sigmoid colon
16 of 45
Questions
resection for cancer, a 71 yr old man is noted to be short of
Notes breath and tachycardic. His vitals are within normal limits with
the exception of temperature of 37.6 °C. Fine crackles are heard
at the bases of both lungs. His medical history is significant for
hypertension, hyperlipidemia, and carotid stenosis. He has
smoked a pack of cigarretes for the past thirty years. A
portable chest radiograph is taken.

Which one of the following is most likely to account for the


patient’s postoperative symptoms?

atelectasis

pneumonia

pulmonary embolism

myocardial infarction
Click on image to Zoom
pneumothorax

Explanation

The correct answer is choice A

Atelectasis is defined as the lack of gas exchange within alveoli, due to alveolar collapse or
fluid consolidation. It may affect part or all of one lung. It is a condition where the alveoli are
deflated, as distinct from pulmonary consolidation.
It is a very common finding in chest x-rays and other radiological studies. It may be caused by
normal exhalation or by several medical conditions. Although frequently described as a
collapse of lung tissue, atelectasis is not synonymous with a pneumothorax, which is a more
specific condition that features atelectasis. Acute atelectasis may occur as a post-operative
complication or as a result of surfactant deficiency. In premature neonates, this leads to infant
respiratory distress syndrome.

Atelectasis may be an acute or chronic condition. In acute atelectasis, the lung has recently
collapsed and is primarily notable only for airlessness. In chronic atelectasis, the affected area
is often characterized by a complex mixture of airlessness, infection, widening of the bronchi
(bronchiectasis), destruction, and scarring (fibrosis)

Acute atelectasis is a common postoperative complication, especially after chest or abdominal


surgery. The most common cause is post-surgical atelectasis, characterized by splinting,
restricted breathing after abdominal surgery. Smokers and the elderly are at an increased risk.
Acute atelectasis may also occur with an injury, usually to the chest (such as that caused by a
car accident, a fall, or a stabbing). Atelectasis following surgery or injury involves most alveoli
in one or more regions of the lungs. In these circumstances, the degree of collapse among
alveoli tends to be quite consistent and complete. Large doses of opioids or sedatives, tight
bandages, chest or abdominal pain, abdominal swelling (distention), and immobility of the
body increase the risk of acute atelectasis following surgery or injury, or even spontaneously.

In acute atelectasis that occurs because of a deficiency in the amount or effectiveness of


surfactant, many but not all alveoli collapse, and the degree of collapse is not uniform.
Atelectasis in these circumstances may be limited to only a portion of one lung, or it may be
present throughout both lungs. When premature babies are born with surfactant deficiency,
they always develop acute atelectasis that progresses to neonatal respiratory distress
syndrome. Adults can also develop acute atelectasis from excessive oxygen therapy and from
mechanical ventilation.

pneumonia (choice B) is an inflammatory condition of the lung. It is often characterized as


including inflammation of the parenchyma of the lung and abnormal alveolar filling with fluid
(consolidation and exudation). Although pneumonia is a postoperative complication and a
cause of postoperative fever, patients will present with infiltrate on chest x-ray with fever >
37.8 °C (100.0 °F), tachycardia, rales/crackles, and decreased breath sounds.

pulmonary embolism (choice C) is a blockage of the main artery of the lung or one of its
branches by a substance that has travelled from elsewhere in the body through the
bloodstream (embolism). Usually this is due to embolism of a thrombus (blood clot) from the
deep veins in the legs, a process termed venous thromboembolism. A small proportion is due
to the embolization of air, fat or amniotic fluid. The obstruction of the blood flow through the
lungs and the resultant pressure on the right ventricle of the heart leads to the symptoms and
signs of PE. On chest radiograph, PE manifests as a a wedge shaped, pleural based
consolidation associated with pulmonary infarction (Hampton's hump) or a focus of oligemia
(vasoconstriction) seen distal to a pulmonary embolus (Westermark Sign).

myocardial infarction (choice D) , commonly known as a heart attack, is the interruption of


blood supply to part of the heart, causing heart cells to die. MI can be a common cause of
postoperative chest pain and shortness of breath. However, patients with an acute myocardial
infarction may have a completely normal chest radiograph.

pneumothorax (choice E) is a collection of air or gas in the pleural cavity of the chest between
the lung and the chest wall. Medical procedures of the chest (iatrogenic), such as the taking of
biopsy samples from lung tissue, inserting a central venous catheter into one of the chest
veins, may lead to injury to the lung and resultant pneumothorax. Traditionally a plain
radiography of the chest, ideally with the X-ray beams being projected from the back
(posteroanterior or PA), has been the most appropriate first investigation. The size of the
pneumothorax, i.e. the amount of space in the chest taken up by free air rather than air-
containing lung, can be determined with a reasonable degree of accuracy by measuring the
distance between the chest wall and the lung.
QUESTION A 34 year old black girl has her right breast removed because
17 of 45
Questions
of a large mass that has been growing for over 10 years. The
Notes tumor weighs 1400g and is found to have a bulging, very firm,
lobulated surface with a whorl like pattern.

This neoplasm is most likely which of the following?

cystosarcoma phylloides

intraductal carcinoma

Duct ectasia

fibroadenoma

Granulomatous mastitis
Explanation

The correct answer is choice D

Fibroadenomas of the breast are small, solid, rubbery, noncancerous, harmless lumps
composed of fibrous and glandular tissue. Unlike typical lumps from breast cancer,
fibroadenomas are easy to move, with clearly defined edges. A fibroadenoma is usually
diagnosed through clinical examination, ultrasound or mammography, and often a needle
biopsy sample of the lump

Fibroadenomas arise in the terminal duct lobular unit of the breast. They are the most
common breast tumor in adolescent women. They also occur in a small number of post-
menopausal women. Their incidence declines with increasing age, and they generally appear
before the age of thirty years, probably partly as a result of normal estrogenic hormonal
fluctuation. Although fibroadenoma is considered a neoplasm, some authors believe
fibroadenoma arises from hyperplasia of normal breast lobule components.

The typical case is the presence of a painless, firm, solitary, mobile, slowly growing lump in
the breast of a woman of childbearing years. In the male breast, fibroepithelial tumors are
very rare, and are mostly Phyllodes tumors.

Most fibroadenomas are left in situ and monitored by a doctor, or the patient in question.
Some are treated by surgical excision. They are removed with a small margin of normal breast
tissue if the preoperative clinical investigations are suggestive of the diagnosis.

Because needle biopsy is often a reliable diagnostic investigation, some doctors may decide
not to operate to remove the lesion, and instead opt for clinical follow-up to serially observe
the lesion over time using clinical examination and mammography to determine the rate of
growth, if any, of the lesion. A growth rate of less than sixteen percent per month in women
under fifty years of age, and a growth rate of less than thirteen percent per month in women
over fifty years of age have been published as safe growth rates for continued non-operative
treatment and clinical observation. Fibroadenomas have not been shown to recur following
complete excision or transform into phyllodes tumours following partial or incomplete excision.
cystosarcoma phylloides (choice A) is typically a large, fast growing mass that forms from the
periductal stromal cells of the breast. They account for less than 1% of all breast neoplasms.
This is predominantly a tumor of adult women, with very few examples reported in
adolescents. Patients typically present with a firm, palpable mass. These tumors are very fast
growing, and can increase in size in just a few weeks. Occurrence is most common between
the ages of 40 and 50, prior to menopause. This is about 15 years older than the typical age of
patients with fibroadenoma, a condition with which Phyllodes tumors may be confused.

intraductal carcinoma (choice B) is the most common type of breast cancer in women. It
comes in two forms: invasive ductal carcinoma(IDC), an infiltrating, malignant and abnormal
proliferation of neoplastic cells in the breast tissue, or ductal carcinoma in situ (DCIS), a
noninvasive, possibly malignant, neoplasm that is still confined to the milk ducts (lactiferous
ducts), where breast cancer most often originates. On physical examination, this lump usually
feels much harder or firmer than benign breast lesions such as fibroadenoma.

Duct ectasia (choice C) is a condition in which there is an obstruction of the lactiferous duct.
Mammary duct ectasia can mimic breast cancer. It is a disorder of premenopausal age. Signs
of duct ectasia can include nipple retraction, inversion, pain, and sometimes bloody discharge.
Histologically, dilation of the large duct is prominent. Pathogenesis may be a reaction to
stagnant colostrum.

Granulomatous mastitis (choice E) are multinucleated giant cells and epithelioid histiocytes
around lobules. Often minor ductal and periductal inflammation is present. The lesion is in
some cases very difficult to distinguish from breast cancer. Patients usually present with a
distinct firm mass mostly in the subareolar region. It occurs on average 2 years and almost
exclusively up to 6 years after pregnancy, usual age range is 17 to 42 years. Use of hormonal
contraceptives, prolactin raising medications and hyperprolactinemia have been implicated in
the pathogenesis or as predisposing factors.
QUESTION A 12 year old boy is brought into your office by his mother. She
18 of 45
Questions
recently noticed a lump on his neck, which the boy admitted he
Notes first felt about nine months ago and seems to be getting larger.
Physical examination reveals a palpable, painless mass of
about 2 cm, located just anterior to the trachea in the midline.
A fine needle aspirate of the mass yields only clear, mucoid
fluid.

Which of the following is the most likely diagnosis?


Parathyroid cyst

Thyroglossal duct cyst

Follicular adenoma

Nodule of multimodal goiter

Lymph node metastasis of follicular carcinoma

Branchial cleft cysts

Dermoid cyst

Explanation

The correct answer is choice B.

A thyroglossal cyst is a fibrous cyst that forms from a persistent thyroglossal duct. It usually
presents as a midline neck lump (in the region of the hyoid bone) that is usually painless,
smooth and cystic, if infected pain can occur. Typically, the cyst will move upwards on
protrusion of the tongue, given its attachment to the embryonic duct. Treatment for a
thyroglossal cyst is surgical resection, often requiring concomitant removal of the midsection
of the hyoid bone (Sistrunk procedure), to prevent recurrence. Although generally benign, the
cyst will be removed if the patient exhibits difficulty in breathing or swallowing, or if the cyst is
infected. Even if these symptoms are not present the cyst may be removed to eliminate the
chance of infection or development of a carcinoma, or for cosmetic reasons if there is unsightly
protrusion from the neck.

Parathyroid cysts (choice A) are an uncommon incidental finding at autopsy. They rarely
exceed 1 cm.

Follicular adenomas (choice C) are rare at this age. They would likely appear in right or left
lobe and be palpable off the midline.
Multinodular goiters (choice D) produce diffuse, nodular thyroid enlargement, not a midline
mass.

Thyroid cancers are rare at this age (except with previous radiation exposure), and lymph
node metastases (choice E) are more common from papillary carcinomas.

Branchial cleft cysts (choice F) are congenital epithelial cysts, which arise on the lateral part of
the neck from a failure of obliteration of the second branchial cleft in embryonic development.

A dermoid cyst (choice G) is a cystic teratoma that contains developmentally mature skin
complete with hair follicles and sweat glands, sometimes clumps of long hair, and often
pockets of sebum, blood, fat, bone, nails, teeth, eyes, cartilage, and thyroid tissue. Because it
contains mature tissue, a dermoid cyst is almost always benign. The rare malignant dermoid
cyst usually develops squamous cell carcinoma in adults; in babies and children it usually
develops endodermal sinus tumor.
QUESTION A 5 year old boy has a midline swelling of the neck that moves with
19 of 45
Questions deglutition and is located below the hyoid bone.
Notes
The mass is likely to be which of the following?

Branchial cleft cyst

Cystic hygroma (lymphatic malformation)

Teratoma (dermoid cyst)

Thyroglossal duct cyst

Midline Thyroid cyst


Explanation

The correrct answer is Choice D

Thyroglossal duct cyst (TGDC) is the most common midline neck mass usually
located at or below the level of the hyoid bone. It is formed from remnants of the
thyroglossal duct which may persist after birth. The thyroglossal duct is the hollow
canal through which the developing thyroid gland descends from its origin in the
foramen caecum at the base of the tongue to its location in the anterior triangle of
the neck. The duct usually involutes after birth; however, if this fails to occur, the
tract may remain dormant for years until some stimulus causes cystic dilatation.
TGDC is usually located at or below the level of the hyoid bone. It can occur at
any age, but it is most common before 20 years of age.

Clinically, it presents as a cystic painless mid-line neck mass in the region of the
hyoid bone. It becomes symptomatic if inflamed, resulting in pain and swelling.
Because of its connection with the hyoid bone and the base of the tongue, TGDC
usually move on swallowing and protrusion of the tongue.

Antibiotics may be indicated if the cyst becomes infected. Definitive treatment is


complete surgical excision. Sistrunk’s procedure is the recommended surgical
approach. The operation includes excision of the cyst in continuity with the mid
portion of the body of the hyoid bone and a small block of muscle around the
foramen caecum.

Branchial cleft cyst (Choice A) is an uncommon neck swelling found along the
anterior border of the sternocleidomastoid, usually at the junction of its upper and
middle third. It is formed from remnants of the second branchial cleft which
persist after embryogenesis. It is found in young adults and presents as a
fluctuant mass that doesn’t transilluminate. It does not move on swallowing.
Definitive treatment is by surgical excision. The cyst may become secondarily
infected during an upper respiratory tract infection. In this case, excision should
be delayed to avoid creating a branchial fistula. Care must be taken not to
damage the carotid vessels and internal jugular vein which usually lie deep to the
swelling.

Cystic hygroma (Choice B) results from congenital malformation of the lymphatic


system. It usually presents in early childhood as a painless soft multi-locular
cystic swellings. The swelling is commonly found in the neck and axilla, although
it can occur anywhere. Small lesions require no treatment, but large cervical
lesions may cause airway obstruction. Surgical excision is the best option, but
this can be difficult in the swelling infiltrates surrounding tissue. Use of
sclerosants e.g. lyophilized product of Streptococcus pyogens, is an option of
treatment.

A teratoma (Choice C) is a germ cell tumour derived from pluripotential stem


cells. The tumour usually contains elements of different tissue derived from all
three germ layers. The common sites for teratomas are the ovary and testis. In
infancy, the commonest site is the sacrococcygeus. A rare form of head and neck
teratomas arises primarily from remnants of Rathke's pouch in the sphenoid bone
region, the tongue and lateral part of the neck. Treatment is by surgical careful
surgical excision as recurrence is very common.

Thyroid cyst (Choice E) is a fluid filled sac in the thyroid gland. The diagnosis is
made clinically by observation and palpation. Confirmation is usually by
ultrasound scan. Usually, thyroid cysts are drained by fine needle aspiration, and
the fluid sent for cellular analysis to rule out any occult malignancy.
QUESTION A 5-years old child presented with unilateral painless mass in
20 of 45
Questions
the neck. On examination, the mass was cystic, non-tender
Notes with smooth surface and located in front of and superficial to
the lower third of the anterior border of sternomastoid muscle.
Your provisional diagnosis was a branchial cyst.

A branchial cyst develops from the vestigial remnants of which


of the following:
2nd branchial cleft

3rd branchial cleft

1st branchial cleft

4th branchial cleft

5th branchial cleft


Click on image to Zoom

Explanation

The correct answer is choice A

The branchial apparatus, which is formed during early gestation, consists of six mesodermal
branchial arches separated externally by five ectodermal branchial clefts and internally by five
endodermal branchial pouches. The second arch grows caudally to meet the fifth arch. This
causes the second, third and the fourth clefts to form one cavity called the cervical sinus of
His. Normally this sinus obliterates during the fetal life.

A branchial cyst develops due to failure of obliteration of the second branchial cleft. 95% of
the branchial anomalies are due to abnormalities of this particular cleft.Usually it is presented
as asymptomatic neck cyst but may enlarge and become tender when infected. Patients also
may present with a mass effect such as difficulty in breathing. Complete surgical resection is
the treatment of choice and results in a good prognosis. Complications of surgical treatment
include recurrence, formation of a persistent fistula, and damage to the cranial nerves.
Patients with infected cysts receive a full course of antibiotics before surgery to decrease the
risk of recurrence and persistent fistula.

Answer B is false as the third branchial cleft rarely gives rise to a cyst and in this case, the
cyst usually lies deep to the sternomastoid muscle.

Answer C is false as the first cleft develops into the external auditory meatus and it is not
related to any structures in the neck.

Answer D is false as the cysts arising from the fourth cleft are extremely rare and they are
usually located inside the thyroid gland or within the mediastinum.

Answer E is false as the fifth is almost always a rudimentary structure.


QUESTION An 8 month old present to the hospital with restlessness, refusal to
21 of 45
Questions eat and vomiting. There is a history of passage red currant jelly
Notes stool. On examination, he is dehydrated with abdominal distention. A
sausage shaped mass is palpated on the abdomen.

The most likely diagnosis includes which of the following:

Cyclical vomiting syndrome

Intussusception

Colic

Volvulus

Infant botulism

Explanation

The correct answer is Choice B

Intussusception is a medical condition that occurs when a part of the intestine


invaginates into the lumen of the adjoining intestine. The condition typically
affects infants between 6 - 8 months of age with an incidence of 1/500
children. There are two main types:

 Idiopathic intussusception: This is the more common form. It usually starts at the ileo-
colic junction and is seen mostly in infants and toddlers.

 Enteroenteral intussusception: this can be jejunojejunal, jejunoileal or Ileo-


ileal. It is commoner in older children and has been linked with an infective
aetiology like Adenovirus. This leads to the enlarged Payers Patches
frequently found at surgery. The enlargement of Peyers Patches
are thought to form the axis for the invagination to occur. There is also an
association with conditions like Henoch-Schönlein purpura, Cystic fibrosis
and haematological dyscrasias.

A high index of suspicion is required, as the symptoms are variable. Delay in


diagnosis can be costly. The classic triad of vomiting, abdominal pain and
passage of blood per rectum occurs in only a third of patients. Pain is colicky,
severe and intermittent, occurring every 10 – 15 minutes and lasting for about 2 –
3 minutes. During each spasm, the child draws up the legs. Vomiting is an early
sign which is initially non-billous. However, as the intestinal obstruction
progresses, vomiting becomes bilious. Passage of blood and mucus per rectum
is often a late feature. This is often referred to as red currant jelly stool, the typical
description of the blood stained stool seen in this condition.

On examination, the child is anxious, ill-looking and usually in pain. Abdominal


examination may reveal a sausage-shaped mass in the right upper quadrant of
the abdomen. This may not always be palpable e.g. in an ileo-ileal
intussusception. On rectal examination, the finger is stained with red currant jelly
stool.

Urgent serum electrolytes should be requested to assess dehydration. Plain


abdominal x-ray will show signs of intestinal obstruction. Abdominal ultrasound
may show the Swiss-roll appearance. Contrast studies using barium enema may
show the Stack of Coins sign. Barium enema has been employed as a means of
reducing the intussusception under radiological control. It is however
contraindicated in a suspected case of bowel gangrene or perforation.
Alternatively, hydrostatic reduction with oxygen is used in some centres. There is
a 10% chance of recurrence with this method. Indications for surgical intervention
include;

 Children under 3 months or over 2 years of age

 Clinical signs of peritonitis

 Gross dehydration

 Failure of medical treatment or recurrence.

Some differential diagnoses include;

Cyclical vomiting syndrome (Choice A) which is characterized by recurrent


explosive bouts of vomiting resulting in time lost from school or work. The
disease is commonly seen in children although some cases have been reported
in adults. The aetiology is unknown. Treatment is conservative through provision
of advice on avoiding known triggers, use of anti-emetics, and supportive care
during acute attack.

Infant colic (Choice C) is a condition that usually occurs within the first three
months of life. It is characterized by inconsolable episodes of excessive and
paroxysmal crying and drawing up of legs. There is no vomiting. It occurs mostly
in the evenings and has no identifiable cause. Colic has been described as
paroxysms of crying, lasting >3hours, occurring >3 days in any week for 3 weeks.
Treatment is usually supportive.

Volvulus (Choice D) refers to the complete twisting of a loop of intestine around it


mesenteric origin. Volvulus neonatorum occurs as a result of failure of normal
rotation of the bowel, with the caecum remaining high in the abdomen and a
band across the duodenum which becomes obstructed. Mid-gut volvulus is a
common form which occurs when there is a twisting of the entire mid-gut around
the axis of the superior mesenteric artery. Volvulus is a dangerous condition that
may lead to infarction of the entire mid-gut. Urgent surgical intervention is
required to treat the condition.

Infant botulism (Choice E) is an acute neurological condition that results from


ingestion of the spores of Clostridium botulinum spores either from the
environment or contaminated honey. The spores then germinate in the intestines
and release toxins, which interferes with the baby’s ability to feed, move, or
breathe. It affects infants between 3 weeks to 6 months of age but children
remain susceptible till their first birthday. Most infants recover from the condition
with supportive care with mortality being about 1%.
QUESTION A 2 year old infant presented with asymptomatic nodule over
22 of 45
Questions
the suprasternal notch. It was biopsied and eventually removed
Notes for histological confirmation.

Which of the following is the most accurate diagnosis?

Branchial cleft cyst

Bronchogenic cyst

Midline cervical cleft

Cervical thymic cyst Click on image to Zoom

Thyroglossal cyst

Explanation
The correct answer is B.

A bronchogenic cyst is a congenital anomaly that develops due to


abnormal budding of the foregut during fetal life. It is the second most
common subtype of foregut cysts found in the middle mediastinum. More than 50% of cases
are diagnosed in patients older than 15 years.

In 85% of cases the cyst is located within the mediastinum. In this


case, it presented with an asymptomatic cystic mass at the
suprasternal notch. It may be associated with fever if inflamed. The most
dangerous complication of such cysts is the mass effect:

 Compression of the GI tract can result in dysphagia.

 Compression of the airway, especially if cyst is just below the carina, can result in life-
threatening respiratory distress.

 Compression of the heart and great vessels can result in dysrhythmias and obstruction
of the vena cava.

Answer A is false. A branchial cleft cyst is a congenital anomaly caused by failure of


obliteration of the 2nd branchial cleft. It is presented by a unilateral cyst usually in front of the
anterior border of the sternomastoid muscle.

Answer C is false. Midline cervical cleft is a very rare congenital


anomaly that is thought to be due to abnormal fusion between the 2nd
and 3rd branchial arches. It is presented as a cleft extending from the
suprasternal notch.

Answer D is false. Thymic cyst is a sequestration cyst arising from the thymopharyngeal duct
at any point of its course from the level of the mandible to the chest. However it is an
extremely rare condition especially when compared to the frequency of the bronchogenic
cysts.

Answer E is false. Thyroglossal cyst occurs due to failure of


obliteration of a part of the thyroglossal duct. In most cases it is
present just below the hyoid bone and over the thyroid cartilage. In
very few cases it may be present at the suprasternal notch.
QUESTION A 67 year old man with hepatitis C and liver cirrhosis requires
23 of 45
Questions
an urgent cholecystectomy. Upon routine preoperative lab
Notes testing, he is found to have abnormal coagulation studies due
to hepatic synthetic dysfunction. He requires an urgent
transfusion of fresh frozen plasma to minimize the risk of
bleeding due to surgery.

The optimal timing of this transfusion is which of the following?

the day before surgery

the night before surgery

on call to surgery

intraoperatively

postoperative period

Explanation

The correct answer is choice C

Fresh frozen plasma (FFP) refers to the liquid portion of human blood that has been frozen and
preserved quickly after a blood donation and will be used for blood transfusion. The term "FFP"
is often used to mean any transfused plasma product.

Few specific indications for the use of FFP exist and is optimally delivered right before surgery.
These indications generally are limited to the treatment of deficiencies of coagulation proteins
for which specific factor concentrates are unavailable or undesirable. Indications for the use of
FFP include the following:

FFP is efficacious for treatment of deficiencies of factors II, V, VII, IX, X, and XI when specific
component therapy is neither available nor appropriate. Requirements for FFP vary with the
specific factor being replaced. For example, hemostatic levels of factor IX in a patient with
severe deficiency are difficult to achieve with FFP alone, whereas patients with severe factor X
deficiency require factor levels of about 10 percent to achieve hemostasis and are easily
treated with FFP.

Patients who are anticoagulated with warfarin are deficient in the functional vitamin K
dependent coagulation factors II, VII, IX, and X, as well as proteins C and S. These functional
deficiencies can be reversed by the administration of vitamin K. However, for anticoagulated
patients who are actively bleeding or who require emergency surgery, FFP can be used to
achieve immediate hemostasis.

FFP can be used as a source of antithrombin III in patients who are deficient of this inhibitor
and are undergoing surgery or who require heparin for treatment of thrombosis. FFP may be
beneficial for the treatment of thrombotic thrombocytopenic purpura.

FFP is useful in infants with secondary immunodeficiency associated with severe protein-losing
enteropathy and in whom total parenteral nutrition is ineffectual. FFP also can be used as a
source of immunoglobulin for children and adults with humoral immunodeficiency. However,
the development of a purified immune globulin for intravenous use largely has replaced FFP.

The day before surgery (choice A) is not the optimal time to decrease bleeding risk in a patient
with known coagulopathy. The factors need to be thawed and given shortly before surgery.

The night before surgery (choice B) is not the optimal time to decrease bleeding risk in a
patient with known coagulopathy. The factors need to be thawed and given shortly before
surgery.

Intraoperatively (choice D) is not the optimal time to decrease bleeding risk in a patient with
known coagulopathy.

Postoperative period (choice E) is too late to reverse the coagulopathy and doesn’t address the
risk of intraoperative bleeding.
QUESTION A 60-year-old man with a past medical history of primary
24 of 45
Questions
hyperparathyroidism presents to the emergency department
Notes with nausea, vomiting, constipation, confusion, polyuria, and
generalized weakness. The patient is immediately placed on a
monitor, intravenous (IV) access is obtained, and a Foley
catheter is inserted. Initial laboratory testing reveals a serum
calcium level of 16 mg/dL.

What is the next step in this patient’s management?

IV fluid resuscitation

Bisphosphonates

Calcitonin

Urgent neck and chest CT scan

IV loop diuretic (furosemide)

Explanation
The correct answer is choice A

Hypercalcaemia is an elevated calcium level in the blood ((Normal range: 9-10.5 mg/dL). It
can be an asymptomatic laboratory finding. However, an elevated calcium level is often
indicative of other diseases and a differential diagnosis should be undertaken if it persists. It
can be due to excessive skeletal calcium release, increased intestinal calcium absorption, or
decreased renal calcium excretion. Primary hyperparathyroidism and malignancy account for
about 90% of cases of hypercalcemia.

EKG findings of a shortened QT interval and a widened T wave suggest hypercalcaemia and
can lead to abnormal heart rhythms. Finally, peptic ulcers may also occur. Symptoms are
more common at high calcium blood values (12.0 mg/dLl). Severe hypercalcaemia (above 15–
16 mg/dL) is considered a medical emergency: at these levels, coma and cardiac arrest can
result. Initial therapy consists of IV fluids and diuretics (furosemide) supplemented with
bisphosphonates and calcitonin.

The general mnemonic for remembering the physical symptoms of hypercalcaemia: "groans
(constipation), moans (psychotic noise), bones (bone pain, especially if PTH is elevated),
stones (kidney stones), and psychiatric overtones (including depression and confusion)." Other
symptoms can include fatigue, anorexia, nausea, vomiting, pancreatitis and increased
urination.

The goal of therapy is to treat the hypercalcemia first and subsequently effort is directed to
treat the underlying cause. Initial therapy involves fluids and forced diuresis. Hydration is
needed because many patients are dehydrated due to vomiting or renal defects in
concentrating urine. Increased salt intake also can increase body fluid volume as well as
increasing urine sodium excretion, which further increases urinary calcium excretion. After
rehydration, a loop diuretic such as furosemide can be given to permit continued large volume
intravenous salt and water replacement while minimizing the risk of blood volume overload
and pulmonary edema. In addition, loop diuretics tend to depress renal calcium reabsorption
thereby helping to lower blood calcium levels.

Bisphosphonates (choice B) can be used as additional therapy after hydration.


Bisphosphonates are taken up by osteoclasts and inhibit osteoclastic bone resorption.
Calcitonin blocks bone resorption and also increases urinary calcium excretion by inhibiting
renal calcium reabsorption

Calcitonin (choice C) can be used as additional therapy after hydration. Calcitonin blocks bone
resorption and also increases urinary calcium excretion by inhibiting renal calcium reabsorption

Neck exploration and removal of the enlarged gland (choice D) is not the first line treatment of
an acutely symptomatic hypercalcemic patient. It is the next best step for definitive treatment
after the patient has stabilized.
QUESTION A 34 year old lady presents with progressive dysphagia to both
25 of 45
Questions
solids and liquids. Heartburn is not a feature of her history. A
Notes barium swallow reveals a dilated esophagus with a hold up of
barium and a rapid taper at the cardia.

All of the following statements are true about this condition


except?

incomplete LES relaxation

decreased LES tone

aperistalsis of the esophagus

most common form is primary


achalasia
secondary to conditions such
Chagas disease

Click on image to Zoom

Explanation
The correct answer is choice B

Achalasia is an esophageal motility disorder involving the smooth muscle layer of the
esophagus and the lower esophageal sphincter (LES). It is characterized by incomplete LES
relaxation, increased LES tone (thus making choice B incorrect), and aperistalsis of the
esophagus (inability of smooth muscle to move food down the esophagus) in the absence of
other explanations like cancer or fibrosis.

The main symptoms of achalasia are dysphagia (difficulty in swallowing) and regurgitation of
undigested food. Dysphagia tends to become progressively worse over time and to involve
both fluids and solids. Some achalasia patients also experience weight loss, coughing when
lying in a horizontal position, and chest pain which may be perceived as heartburn. The chest
pain experienced, also known as cardio spasm and Non Cardiac Chest Pain (NCCP) can often
be mistaken for a heart attack. It can be extremely painful in some sufferers. Food and liquid,
including saliva, are retained in the esophagus and may be inhaled into the lungs (aspiration).

The most common form is primary achalasia, which has no known underlying cause. It is due
to the failure of distal esophageal inhibitory neurons. However, a small proportion occurs
secondary to other conditions, such as esophageal cancer or Chagas disease (an infectious
disease common in South America).

Due to the similarity of symptoms, achalasia can be mistaken for more common disorders such
as gastroesophageal reflux disease (GERD),hiatus hernia, and even psychosomatic disorders.
Specific tests for achalasia are barium swallow and esophageal manometry. In
addition,endoscopy of the esophagus, stomach and duodenum (esophagogastroduodenoscopy
or EGD), with or without endoscopic ultrasound, is typically performed to rule out the
possibility of cancer. The internal tissue of the esophagus generally appears normal in
endoscopy, although a "pop" may be observed as the scope is passed through the non-
relaxing lower esophageal sphincter with some difficulty, and food debris may be found above
the LES.

Various treatments are available, although none cure the condition. Certain medications or
Botox may be used in some cases, but more permanent relief is brought by esophageal
dilatation and surgical cleaving of the muscle (Heller myotomy).
QUESTION A six week old male infant presents with a history of projectile non
26 of 45
Questions bilous vomitting that occurs shortly after brest feeding.
Notes
The treatment of choice for the suspected condition includes which
of the following:

vagotomy

pyloroplasty

pyloromyotomy

Heller's myotomy

Nissen fundoplication

Explanation

The correct answer is Choice C

Idiopathic hypertrophic pyloric stenosis (IHPS) is a condition characterized by a


narrowing of the pylorus, which connects the stomach to the duodenum. There is
hypertrophy of the circular muscles at the gastro-duodenal junction, leading to a
reduction in the size of the lumen and obstruction of gastric emptying. The
incidence is about 3/1000 live births and the cause is unknown. It is commoner in
the first-born male children and has a higher incidence in Caucasians compared
to the Black race.

Clinically, IHPS presents in neonates with initial non-bilious vomiting at about the
4th -8th week of life. Vomiting typically occurs shortly after every feeding, with
increasing intensity which eventually becomes projectile. The baby usually
appears hungry and sucks vigorously after every episode of vomiting.
Late presentations and delayed diagnosis gives a clinical picture of dehydration
and shock in an infant with malnutrition and evidence of poor weight gain.

On physical examination, an enlarged pylorus may be felt in the right upper


quadrant, just below the costal margin. This mass is typically described as an
“olive”. The patient must be calm and co-operative for the examiner to appreciate
this finding.

Ultrasound diagnosis is the standard imaging investigation. Barium studies and


upper GI endoscopy also provide useful information to assist in management of
the condition.

The definitive treatment is surgical repair of the stenosis.


Pyloromyotomy (Choice C) is the recommended surgical operation as it gives
excellent results. Ramstedt pyloromyotomy is the preferred approach, via a
transverse right upper quadrant incision that splits the rectus muscle and fascia.

Preoperative management is aimed at correcting dehydration and electrolyte


imbalance. Post-operatively, recovery is usually excellent, and the child may
resume graded oral feeds within 8 hours following the surgery. Discharge
home can follow as soon as the child remains dehydrated and can tolerate oral
feeds.

Pyloroplasty (Choice B) refers to the surgical widening of the pylorus to aid


gastric content emptying. Pyloroplasty is commonly performed with a truncal
vagotomy (Choice A) as part of the treatment for peptic ulcer disease. Vagotomy
is a delicate procedure involving the severance of the trunks of the vagus nerve,
leading to reduced gastric acid secretion.

Nissen Fundoplication (Choice D) is the surgical procedure used to treat gastro-


esophageal reflux disease (GERD) and hiatus hernia. The fundus of the stomach
is wrapped around the lower end of the oesophagus to provide additional
support, thereby preventing the reflux of acid in GERD and the recurrence of a
hiatus hernia.
Heller’s Myotomy (Choice E) is a surgical procedure that involves cutting of the
muscles of the lower oesophageal or cardiac sphincters, allowing passage of
food from the oesophagus into the stomach. It is used in the treatment of
Achalasia.
QUESTION A 45-year-old woman with no significant past medical history
27 of 45
Questions
presents with a firm 3-cm mass in the right thyroid lobe with no
Notes clinical evidence of nodal involvement. She has no family
history of cancers. A fine-needle aspiration (FNA) is performed
and histologic examination confirms a diagnosis of papillary
cancer.

What is the next appropriate step in management?

Reassure and Follow up in 1-Year

Lobectomy and isthmusectomy

Radioiodine ablation

Total thyroidectomy

Ultrasound of thyroid

Explanation

The correct answer is choice D

Papillary thyroid cancer or papillary thyroid carcinoma is the most common type of thyroid
cancer, representing 75% to 85% of all thyroid cancer cases. It occurs more frequently in
women and presents in the 30-40 year age group. It is also the predominant cancer type in
children with thyroid cancer, and in patients with thyroid cancer who have had previous
radiation to the head and neck. It has the most favorable prognosis of all thyroid cancers.

Most often the first symptom of thyroid cancer is a nodule in the thyroid region of the neck.
However, many adults have small nodules in their thyroids, but typically under 5% of these
nodules are found to be malignant. Sometimes the first sign is an enlarged lymph node. Later
symptoms that can be present are pain in the anterior region of the neck and changes in
voice.

Thyroid cancer is usually found in a euthyroid patient, but symptoms of hyperthyroidism or


hypothyroidism may be associated with a large or metastatic well-differentiated tumor.
Thyroid nodules are of particular concern when they are found in those under the age of 20.
The presentation of benign nodules at this age is less likely, and thus the potential for
malignancy is far greater.

Thyroid cancer may require surgery. Common surgeries include thyroidectomy, lobectomy,
and tracheostomy. Surgical treatment is dependent on the size of the disease:

 Minimal disease (diameter up to 1.0 centimeters) - hemithyroidectomy (or unilateral


lobectomy) and isthmectomy may be sufficient.

 Gross disease (diameter over 1.0 centimeters) - total thyroidectomy, and central
compartment lymph node removal is the therapy of choice. Additional lateral neck
nodes can be removed at the same time if an ultrasound guided FNA and thyrobulin TG
cancer washing was positive on the pre-operative neck node ultrasound evaluation.

Lobectomy and isthmusectomy (choice B) is indicated for minimal disease up to 1.0 cm in


diamater.

Radioiodine ablation (choice C) may be used when the cancer is unresectable or recurrence
after resection. Radioactive Iodine-131 is used in patients with papillary or follicular thyroid
cancer for ablation of residual thyroid tissue after surgery and for the treatment of thyroid
cancer. It may also be used to relieve pain from bone metastasis.
Ultrasound of thyroid (choice E) is performed to confirm the presence of a nodule and assess
the status of the whole gland. It is not performed after a fine needle aspiration confirms the
presence of thyroid neoplasm.
QUESTION A previously healthy 65 year old woman presents with jaundice
28 of 45
Questions
that has developed in the past couple months. She was
Notes recently diagnosed with diabetes a month ago. She denies
abdominal pain but her stools have been pale and her urine has
been darker than usual. Upon examination, you see a jaundiced
woman with scleral icterus and a palpable gallbladder.
Abdominal ultrasound reveals gall stones, a 1.7 cm common
bile duct, and a 2x3 cm mass in the head of the pancreas.
Blood tests reveal elevated direct and total bilirubin.

The most appropriate diagnostic test to perform next includes


which of the following?

Endoscopic retrograde cholangiopancreatography (ERCP)

Selective angiography

Contrast CT scan of abdomen and pelvis

Percutaneous transhepatic cholangiography

Magnetic resonance cholangiopancreatography (MRCP)

Explanation

The correct answer is choice C

Pancreatic cancer is a malignant neoplasm of the pancreas. The prognosis is poor, with fewer
than 5% of those diagnosed still alive five years after diagnosis. Complete remission is still
rare. About 95% of exocrine pancreatic cancers are adenocarcinomas. The remaining 5%
include adenosquamous carcinomas, signet ring cell carcinomas, hepatoid carcinomas, colloid
carcinomas, undifferentiated carcinomas, and undifferentiated carcinomas with osteoclast-like
giant cells. Exocrine pancreatic tumors are far more common than pancreatic endocrine
tumors, which make up about 1% of total cases

Pancreatic cancer is sometimes called a "silent killer" because early pancreatic cancer often
does not cause symptoms, and the later symptoms are usually nonspecific and varied.
Therefore, pancreatic cancer is often not diagnosed until it is advanced. Common symptoms
include:

Pain in the upper abdomen that typically radiates to the back (seen in carcinoma of the body
or tail of the pancreas)

 Loss of appetite and/or nausea and vomiting

 Significant weight loss

 Painless jaundice (yellow skin/eyes, dark urine) when a cancer of the head of the
pancreas (about 60% of cases) obstructs the common bile duct as it runs through the
pancreas. This may also cause pale-colored stool and steatorrhea.

 Trousseau sign, in which blood clots form spontaneously in the portal blood vessels,
the deep veins of the extremities, or the superficial veins anywhere on the body, is
sometimes associated with pancreatic cancer.

 Diabetes mellitus, or elevated blood sugar levels. Many patients with pancreatic cancer
develop diabetes months to even years before they are diagnosed with pancreatic
cancer, suggesting new onset diabetes in an elderly individual may be an early
warning sign of pancreatic cancer.

The initial presentation varies according to location of the cancer. Malignancies in the
pancreatic body or tail usually present with pain and weight loss, while those in the head of
the gland typically present with steatorrhea, weight loss, and jaundice. The recent onset of
atypical diabetes mellitus, a history of recent but unexplained thrombophlebitis (Trousseau
sign), or a previous attack of pancreatitis are sometimes noted. Courvoisier sign defines the
presence of jaundice and a painlessly distended gallbladder as strongly indicative of pancreatic
cancer, and may be used to distinguish pancreatic cancer from gallstones. Tiredness,
irritability and difficulty eating because of pain also exist. Pancreatic cancer is often discovered
during the course of the evaluation of aforementioned symptoms.

Liver function tests can show a combination of results indicative of bile duct obstruction (raised
conjugated bilirubin, γ-glutamyl transpeptidase and alkaline phosphatase levels). CA19-9
(carbohydrate antigen 19.9) is a tumor marker that is frequently elevated in pancreatic
cancer. Imaging studies, such as computed tomography (CT scan) and endoscopic ultrasound
(EUS) can be used to identify the location and form of the cancer.

Endoscopic retrograde cholangiopancreatography (choice A) is a diagnostic and therapeutic


procedure that uses endoscopy and fluoroscopy to access the common bile duct and the
billiary tree. A endoscopic stent can be deployed in the common bile duct or to retrieve a stone
in order to relieve obstructive jaundice. Since the source of obstruction is a pancreatic mass
and the common bile duct is not dilated, there is no role for ERCP in this case.

Selective angiography (choice B) is not indicated for pancreatic masses. Angiography is a


medical imaging technique used to visualize blood vessels. This is traditionally done by
injecting a radio-opaque contrast agent into the blood vessel and imaging using X-ray based
techniques such as fluoroscopy.

Percutaneous transhepatic cholangiography (PTC) (choice D) is a radiologic technique used to


visualize the anatomy of the biliary tract. A contrast medium is injected into a bile duct in the
liver, after which X-rays are taken. It allows access to the biliary tree in cases where
endoscopic retrograde cholangiopancreatography (ERCP) has been unsuccessful. Since the
source of billiary obstruction is outside the billiary tree, it is not indicated.

Magnetic resonance cholangiopancreatography (choice E) is a medical imaging technique that


uses magnetic resonance imaging to visualise the biliary and pancreatic ducts in a non-
invasive manner. This procedure can be used to determine if gallstones are lodged in any of
the ducts surrounding the gallbladder. In the absence of ductal dilation, this is not the initial
study to assess masses in the pancreas.
QUESTION A newborn infant presented with repeated bilious vomiting,
29 of 45
Questions
delayed passage of meconium, scaphoid abdomen with
Notes epigastric fullness. An erect plain x-ray film was done revealing
double-bubble sign. The case was diagnosed as duodenal
atresia.

Duodenal atresia is most likely to occur in which of the


following syndrome:

Down syndrome

Cushing's syndrome

Turner's syndrome

Klinefelter's syndrome

all of the above


Click on image to Zoom

Explanation

The correct answer is A.

Approximately 30% of cases of duodenal atresia are associated with


Down syndrome.

Down syndrome (Triosomy 21) is a chromosomal disorder caused by


presence of extra 21st chromosome. The typical phenotype of the
Down syndrome includes mental retardation, characteristic facial
features, hand anomalies and congenital heart defects.

Answer B is false. Cushing syndrome is caused by prolonged


exposure to high levels of glucocorticoids whether endogenous or
exogenous. Its main criteria include muscle weakness, weight gain,
hirsutism, hypertension, diabetes mellitus and diminished immunity.
Answer C is false. Turner syndrome is a chromosomal disorder caused
by absence of one x-chromosome in a female. The karyotype is 45,x0. The typical
phenotype includes short stature, absence of breast development of breasts, high-arched
palate and webbed neck.

Answer D is false. Klinefelter syndrome is a chromosomal disorder caused by presence of


extra sex chromosome. The karyotype is 47,xxy. The typical phenotype includes
hypogonadism and gynecomastia in males.
QUESTION A 55-year-old, obese but previously healthy woman is
30 of 45
Questions
hospitalized for pneumonia. On the 15th hospital day she is
Notes found to have swelling and tenderness of her right leg which
apparently had developed over the past 48 hours. On her limb
examination, straight leg raises elicits pain in the calf muscles.
Her leg is red and warm, but no fever is present. Cardiac, and
abdominal examinations are absolutely normal, except the
positive findings for pneumonia on respiratory examination.

An ultrasound examination reveals findings suggestive of


femoral vein thrombosis. She has no previous history of
abortions, but she has been using post menopausal hormonal
replacement therapy for the last year.

Which of the following conditions is most likely to have


contributed to the appearance of these findings?

Trousseau syndrome

Factor v leiden mutation

Warfarin induced skin necrosis

Disseminated intravascular coagulation,DIC

Prolonged immobilization
Explanation

The correct answer is choice E.

Among many causes of femoral vein thrombosis (deep vein thrombosis, DVT), the most simple
cause is prolonged immobilization. All of the other mentioned causes are related to DVT but
not the best answer in this case.

DVT's are a medical emergency, as there is a 3% chance of a pulmonay embolism that can be
fatal in some cases. Causes and risk factors for DVT's include the following:

 An inherited condition, factor V Leiden deficiency causes increased risks for clotting
(factor V Leiden deficiency is most common type of genetic thrombophilia)

 Cancer and its treatment (Trousseau syndrome)

 Low blood flow in a deep vein due to injury, surgery, immobilization due to
hospitalization or bed ridden patients

 Sitting for a long time, such as long trips in a car or on an airplane

 Pregnancy, especially the first six weeks after giving birth (postpartum)

 Taking birth control pills or hormone therapy, including postmenopausal HRT

 Other drugs such as estrogen or erythropoietin

 Other medical conditions, such as varicose veins


 Over the age of 60 (although deep vein thrombosis can occur in any age group)

 Obesity, diabetes and smoking

Prolonged immobilization (choice E) can cause circulatory stasis, which is one of the
predisposing factors for DVT described by Virchow in his famous triad:

 endothelial injury

 stasis of circulation

 hypercoagulability

For travelers, prolonged immobilization is defined as a flight longer than 6-10 hours (most
studies use an average of 8 hours). For nursing home residents, the definition of prolonged
immobilization varies between less than 15 days (increased risk of DVT) and/or longer than 3
months.

In certain cancers, especially adenocarcinomas of the pancreas and lung, associated venous
thrombosis and hypercoagulability is known as trousseau sign (choice A). When
thrombophlebitis occurs repeatedly in different locations, it is known as "migrating
thrombophlebitis".

Factor V Leiden deficiency (choice B) is a type of thrombophilia that occurs due to activated
protein C resistance. APC's main role is to degrade Factor Va. APC resistance is the inability of
protein C to degrade Factor Va. This may be hereditary or acquired in some cases. Suspicion
of factor V Leiden deficiency should be considered in any patient below the age of 45, or in any
person with a family history of venous thrombosis. Factor V Leiden deficiency doubles the life-
time risk of developing a DVT in an individual.

Warfarin induced skin necrosis (choice C), is a type of thrombophilia which occurs due to
protein C deficiency. This may be inherited or acquired. The skin lesions of WISN occur on the
extremities, torso, breasts, and penis. They begin as erythematous macules and, if appropriate
therapy is not initiated promptly, they may evolve to become purpuric and necrotic bullae.

Disseminated intravascular coagulation, DIC (choice D) is another type of thrombophilia. DIC


may be congenital or acquired. It is characterized by a systemic activation of the blood
coagulation system, which results in the generation and deposition of fibrin, leading to
microvascular thrombi in various organs and contributing to the development of multiorgan
failure. Later on in the natural course of DIC, a bleeding disorder may also develop.
QUESTION A healthy female infant is born via spontaneous vaginal delivery at
31 of 45
Questions 38 weeks gestation, weighing 3.5kg with an Apgar score of 7 and 9
Notes at one and five minutes respectively. Twenty four hours after birth,
the mother is concerned that the infant has failed to pass meconium.
The infant is afebrile and breast feeding appropriately.

What is the appropriate initial management?

IVF and nasogastric tube aspiration

Defunctioning colostomy

Reassure mother and continue observation for another 24


hours

Posterior saggital anorectoplasty

Lateral Invertogram

Stool Enema or suppository

Explanation

The correct answer is choice C


The mother should be reassured as it is normal for passage of meconium to be delayed up to
48 hours after birth. However, further delay will require an investigation of the infant. A few
possible causes of delayed passage of meconium include Imperforate anus, Hirschsprungs
disease and Meconium Ileus.

Imperforate anus refers to a congenital anomaly resulting from failure of complete development and
canalization of the ano-rectal canal. It affects about 1 in 4000 new births; there is equal prevalence in males
and females. The clinical forms of imperforate anus include rectal atresia which may be high or low,
depending on the position of the deformity either above or below the levator ani muscle. Low lesions are
usually associated with rectovesical, recto-urethral or recto-vaginal fistulae. These associated anomalies
result in bladder and sexual dysfunction. Clinically, infants usually present with intestinal obstruction
manifesting as failure to pass meconium. The extent of the anal defect or rectal atresia can be confirmed
using a lateral invertogram (Choice E). The infant is inverted and the anus is marked with a radio-opaque
marker and a lateral x-ray taken. Air in the rectum will rise to the highest point of the rectum that is patent
indicating the extent of the atresia. Definitive treatment is surgical reconstruction. The preferred method is
the posterior saggital anorectoplasty (PSARP) (Choice D), usually for low defects. High defects are usually
managed initially with a colostomy (Choice B) and subsequent surgical intervention.

Meconium ileus is a clinical condition result in from inspissated meconium blocking the small bowel of
neonates. It is frequently associated with Cystic Fibrosis as 15% of patients with this condition present with
meconium ileus. Lack of pancreatic enzymes and loss of intestinal mucus leading to the formation of
thickened tenacious meconium which causes intraluminal obstruction in the terminal ileum. Infants with
meconium ileus usually present in the first few hours of life with abdominal distention, bile stained vomitus
and failure to pass meconium. Diagnosis is usually clinical, but can be aided by an abdominal radiograph
which shows a mottled soap-bubble appearance of meconium in the right iliac fossa due to the presence of
fat droplets. Treatment of the condition requires relieving of the intestinal obstruction using gastrograffin
enema or laparotomy in complicated cases. Concurrent management of CF is indicated if diagnosed.

Hirschsprungs disease is another congenital condition that presents with failure to pass meconium in the first
few days of life. It is caused by failure of ganglion cells of the myenteric and submucosal plexuses to migrate
into the hindgut, resulting in failure of coordinated peristalsis in the distal aganglionic segment of the gut.
75% of cases are confined to the rectosigmoid colon (short segment disease), while 10 % of cases involve
the entire colon (long segment disease). Infants present with features of intestinal obstruction abdominal
detention, bile-stained vomiting and failure to pass meconium in the first few days of life. Diagnosis is
confirmed by suction rectal biopsy, which demonstrates the absence of ganglion cells. Anal manometry and
contrast studies may help determine the extent of gut involvement. Definitive management is surgical,
requiring an initial defunctioning colostomy (Choice B) and a subsequent pull-through surgery, anastomosing
the normal bowel to the anus.
QUESTION A 24-year-old woman presents with severe diffuse abdominal
32 of 45
Questions
pain. She has a six-month history of crampy abdominal pain
Notes accompanied by intermittent bloody diarrhea. On physical
exam she is found to have marked abdominal distention.
Further workup reveals toxic megacolon and she undergoes an
emergent colectomy. After the operation, she is brought to the
SICU and the decision is made to feed her through total
parental nutrition (TPN).

Which one of the following constituents in your TPN bag is


MOST likely to INCREASE stupor?

Glucose

Leucine

Pantothenic acid

L-Tryptophan

Lipids

Explanation

The correct answer is choice D

Parenteral nutrition (PN) is feeding a person intravenously, bypassing the usual process of
eating and digestion. The person receives nutritional formulas containing salts, glucose, amino
acids, lipids and added vitamins. It is called total parenteral nutrition (TPN) when no food is
given by other routes.

Parenteral nutrition is provided when the gastrointestinal tract is nonfunctional because of an


interruption in its continuity or because its absorptive capacity is impaired. It has been used
for comatose patients, although enteral feeding is usually preferable, and less prone to
complications. Indications: TPN may be the only feasible option for patients who do not have a
functioning GI tract or who have disorders requiring complete bowel rest, such as the
following: Crohn's disease or ulcerative colitis, bowel obstruction, certain pediatric GI
disorders, e.g., congenital GI anomalies, prolonged diarrhea regardless of its cause, or short
bowel syndrome due to surgery.

The most common complication of PN use is bacterial infection, usually due to the increased
infection risk from having an indwelling central venous catheter. In patients with frequent
bacterial infections, fungal infections can also occur.

Total parenteral nutrition increases the risk of acute cholecystitis due to complete unusage of
gastrointestinal tract, which may result in bile stasis in the gallbladder. Other potential
hepatobiliary dysfunctions include steatosis, steatohepatitis, cholestasis, and cholelithiasis.
Such complications are suggested to be the main reason for mortality in people requiring long-
term total parenteral nutrition, such as in short bowel syndrome.

Tryptophan is one of the 20 standard amino acids, as well as an essential amino acid in the
human diet. In addition, tryptophan functions as a biochemical precursor for serotonin and
niacin. Once inside the CNS, tryptophan is converted into serotonin in the raphe nuclei by the
normal enzymatic pathway. The resultant serotonin is further metabolised into melatonin by
the pineal gland. Hence, tryptophan causes drowsiness by sleep-promoting melatonin in the
brain

Glucose (choice A) in TPN is known to cause liiver failure, often related to non-alcoholic
steatohepatitis (NASH). Liver dysfunction can be limited to a reversible cholestatic jaundice
and to fatty infiltration (demonstrated by elevated transaminases) by restricting glucose or
adding insulin to the TPN mixture. Severe hepatic dysfunction is a rare complication.

Leucine (choice B) causes delirium, neurologic compromise, and can be life-threatening if


given in excess in TPN.

Pantothenic acid (choice C) is a water-soluble vitamin. Pantothenic acid is used to synthesize


coenzyme-A (CoA), and well as to synthesize and metabolize proteins, carbohydrates, and
fats.

Excess pantothenic acid is widely known to cause nausea, headaches, diarrhea and a lack of
energy. The lack of energy is believed to be the depleted vitamin B12 (cobalamin), as massive
amounts of vitamin B5 will deplete other vitamin B components. However, it does not cause
stupor.
Lipids (choice E) that are given in TPN infusions are known to cause venous thrombosis and
rarely priapism.
QUESTION A 60 year old man presents for followup after being discharged
33 of 45
Questions
from the hospital with a blood clot in his left leg a week ago.
Notes Upon further questioning, he has had a history of painless
jaundice for the past 6 months. He was recently diagnosed with
diabetes a month ago. He mentions that his stools have been
pale and his urine has been darker than usual since his last
visit. Upon inspection, you see jaundiced man with scleral
icterus.

Which of the following would be the next appropriate


management step?

cholecystojejunostomy

percutaneous transhepatic stent

endoscopic stent

endoscopic papillotomy

chemotherapy

Explanation

The correct answer is choice A

Pancreatic cancer is a malignant neoplasm of the pancreas. The prognosis is poor, with fewer
than 5% of those diagnosed still alive five years after diagnosis. Complete remission is still
rare. About 95% of exocrine pancreatic cancers are adenocarcinomas. The remaining 5%
include adenosquamous carcinomas, signet ring cell carcinomas, hepatoid carcinomas, colloid
carcinomas, undifferentiated carcinomas, and undifferentiated carcinomas with osteoclast-like
giant cells. Exocrine pancreatic tumors are far more common than pancreatic endocrine
tumors, which make up about 1% of total cases

Pancreatic cancer is sometimes called a "silent killer" because early pancreatic cancer often
does not cause symptoms, and the later symptoms are usually nonspecific and varied.
Therefore, pancreatic cancer is often not diagnosed until it is advanced. Common symptoms
include:

 Pain in the upper abdomen that typically radiates to the back (seen in carcinoma of the
body or tail of the pancreas)

 Loss of appetite and/or nausea and vomiting

 Significant weight loss

 Painless jaundice (yellow skin/eyes, dark urine) when a cancer of the head of the
pancreas (about 60% of cases) obstructs the common bile duct as it runs through the
pancreas. This may also cause pale-colored stool and steatorrhea.

 Trousseau sign, in which blood clots form spontaneously in the portal blood vessels,
the deep veins of the extremities, or the superficial veins anywhere on the body, is
sometimes associated with pancreatic cancer.

 Diabetes mellitus, or elevated blood sugar levels. Many patients with pancreatic cancer
develop diabetes months to even years before they are diagnosed with pancreatic
cancer, suggesting new onset diabetes in an elderly individual may be an early
warning sign of pancreatic cancer.

Treatment of pancreatic cancer is mainly surgical and depends on the stage of the cancer. The
Whipple procedure is the most common surgical treatment for cancers involving the head of
the pancreas. This procedure involves removing the pancreatic head and the curve of the
duodenum together (pancreato-duodenectomy), making a bypass for food from stomach to
jejunum (gastro-jejunostomy) and attaching a loop of jejunum to the cystic duct to drain bile
(cholecysto-jejunostomy). It can be performed only if the patient is likely to survive major
surgery and if the cancer is localized without invading local structures or metastasizing. It can,
therefore, be performed in only the minority of cases.
percutaneous transhepatic stent ( choice B) may be used to drain bile until a more permanent
solution for the obstruction is performed (e.g. surgery). Additionally, self expanding metal
stents can be placed across malignant biliary strictures to allow palliative drainage.
Percutaneous placement of metal stents can be utilised when therapeutic ERCP has been
unsuccessful, anatomy is altered precluding endoscopic access to the duodenum, or where
there has been separation of the segmental biliary drainage of the liver, allowing more
selective placement of metal stents. It is generally accepted that percutanous biliary
procedures have higher complication rates than therapeutic ERCP. Complications encountered
include infection, bleeding and bile leaks.

endoscopic stent (choice C) is a metallic tube used to hold open a structure in the
gastrointestinal tract in order to allow the passage of food, chyme, stool, or other secretions
required for digestion. Stents are inserted by endoscopy, wherein a fibre optic camera is
inserted either through the mouth or retrograde through the colon, in order to reach an area
of narrowing.

The vast majority of stents are used to alleviate symptoms caused by cancers of the
gastrointestinal tract that obstruct the interior of the tube-like (or luminal) structures of the
bowel — namely the esophagus, duodenum, common bile duct and colon. Biliary stents are
used to palliatively treat tumours of the pancreas or bile duct that obstruct the common bile
duct. They are inserted at the time of ERCP, a procedure that uses endoscopy and fluoroscopy
to access the common bile duct. Stents are not the first line management of pancreatic head
cancers.

endoscopic papillotomy (choice D) is used to relieve stenosis of the sphincter of Oddi, a


muscular valve, that prevents the opening and release of bile or pancreatic fluids into the
duodenum in response to food entering the duodenum. Obstructions are commonly caused by
stones or scarring. Obstruction of the valve can cause pancreatic pain, jaundice, attacks of
pancreatitis. Endoscopic papillotomy is not the first line management of pancreatic head
cancers.

chemotherapy ( choice E) is the next appropriate step after pancreatic head resection. After
surgery, adjuvant chemotherapy with gemcitabine has been shown in several large
randomized studies to significantly increase the 5-year survival (from approximately 10 to
20%), and should be offered if the patient is fit after surgery.
QUESTION Meckel's diverticulum is the most common congenital anomaly
34 of 45
Questions
of small intestine. It is present in about 2% of population
Notes located 50-60 cm proximal to the ileo-cecal valve. It is cased by
incomplete obliteration of the vitelline duct.

Regarding Meckel’s diverticulum, all of the following


statements are correct except:

When hemorrhage
complicates a Meckel’s
diverticulum, blood
transfusion is often necessary
Bleeding from a Meckel’s
diverticulum is the
commonest cause of GI
hemorrhage in the pediatric
population
Meckel’s diverticulum is the
commonest cause in pediatric
cases of intussusception.
Meckel’s diverticula do not
contain ectopic tissue.
Click on image to Zoom
All of the above are false

All of the above are true

Explanation

The correct answer is choice E.

None of these phrases is true.

Answer A is a false phrase. Meckel’ diverticulum often contains


ectopic gastric tissue the secretes gastric acid leading to ulceration of
the surrounding tissues. The bleeding from these ulcers is the
commonest presentation of this case but it is usually not severe and
does not require blood transfusion.

Answer B is false as the commonest causes of GI bleeding in


pediatrics include oesophagitis, gastritis, duodenitis, intussusception,
colonic polyp, and anorectal disorders rather than Meckel’s
diverticulum.

Answer C is false as the commonest leading point of intussusception


is enlarged Peyer patches due to adenovirus infection (95% of cases).

Answer D is false as heterotropic gastric mucosa was found in 62% of


cases, pancreatic tissue was found in 6%, both pancreatic tissue and
gastric mucosa were found in 5%, jejunal mucosa was found in 2%.
Rarely, colonic, rectal, endometrial, and hepatobiliary tissues have
been noted.

Suggested References
Elsayes KM, Menias CO, Harvin HJ, Francis IR. Imaging manifestations of Meckel's
diverticulum. AJR Am J Roentgenol. Jul 2007;189(1):81-8.
QUESTION A 3 week old baby is brought to the emergency department
35 of 45
Questions
with a 36 hour history of progressive vomiting. The birth weight
Notes was 3.1 kg. On physical examination, his weight is 2.8 kg; he
has at least 5% dehydration to date and is very irritable but not
toxic. His blood work reveals:

 pH: 750
 paCO2: 30 mm/Hg
 HCO3: 31 mmol/L
 Base excess; 7 mmol/L
 Potassium: 3.0 mmol/L
 Sodium: 135 mmol/L
 Chloride: 70 mmol/L
 Urea nitrogen 7.2 mmol/L

Which one of the following diagnoses is the most likely?

obstructive uropathy

gastroenteritis

pyloric stenosis

congenital adrenal hyperplasia

cystic fibrosis

Explanation

The correct answer is choice C.

Pyloric stenosis involves hypertrophy of the circular muscle of the


pylorus, resulting in narrowing and obstruction of the pyloric channel
by compression of longitudinal folds of mucosa. Gastric outlet
obstruction results in emesis, which is characteristically nonbilious and
projectile. Protracted emesis, as well as failure of the stomach to
empty into the duodenum, results in progressive dehydration (explains
the weight loss), electrolyte abnormalities, acid-base disorders, weight
loss, and, potentially, shock.

Gastric fluid loss is associated with the loss of H and Cl ions


(diminished blood Cl) leading to metabolic alkalosis (elevated pH,
diminished PaCO2, increased HCO3 and increased base excess).
Urinary Na and HCO3 losses, which compensate for Cl losses, increase
this alkalosis. However, urinary excretion of K and H increases in an
attempt to preserve Na and blood volume (normal Na level and
diminished K level)

Answer A, obstructive uropathy, is false as the renal function is


normal (normal BUN). Also there would be fluid retention leading to
weight gain and not weight loss. Obstructive uropathy cannot also
explain the repeated vomiting.

Answer B is false as gastroenteritis would not produce such


electrolyte disturbances unless in severe cases. Also the patient
doesn't’ look toxic.

Answer D, congenital adrenal hyperplasia, is false as in this disease


there would be hyponatremia and hyperkalemia.

Answer E is false as cystic fibrosis is not usually presented with such


electrolyte disturbances. Also it is usually presented with respiratory
distress along with manifestations of intestinal obstruction.
QUESTION You are seeing a 52 year old male patient in the office who
36 of 45
Questions
smokes three packs of cigarettes per day and has smoked for
Notes over 35 years. He has lost 15 lbs during the past four months
without being on a diet. He also complains about on going
shortness of breath for the past 2 months. His symptoms seem
to worsen despite the bronchodilators that you have
prescribed. He has a history of chronic cough but rarely
productive.

A few days later, he presented to the emergency due to


worsening shortness of breath. On physical examination, he
was tachypneic with bilateral upper extremity swelling along
with facial puffiness. There was no evidence of cyanosis and
his oxygen saturation was 92% on 2L oxygen. His respiratory
exam was normal and cardiac exam showed normal heart
sounds a and JVP was approximately 2 cm above the sternal
angel. His abdomen was soft and nontender.

The most likely cause of these findings is which of the


following:

congestive heart failure

chronic obstructive pulmonary disease (COPD) and


emphysema

obstruction of the superior vena cava

pulmonary embolus

thrombosis of the subclavian vein

Explanation

The correct answer is choice C.

More than 90% of patients who present with superior vena cava syndrome, as in this patient,
have an associated malignancy (causing weight loss).

Superior vena cava (SVC) syndrome (choice C) is characterized by gradual


compression/obstruction of the superior vena cava, with a gradual increase in
symptomatology. The thin walled superior vena cava is surrounded by rigid structures and is
relatively easy to compress. Low intravascular pressure also allows for the possibility of
thrombus formation.
SVCS is chiefly associated with malignancies, as the Infectious causes (e.g. syphilis,
tuberculosis) have decreased because of improvements in antibiotic therapy. Commonly
associated malignancies include bronchogenic carcinoma among other lung cancers,
lymphomas, and metastatic malignant diseases. Clinical presentations with a history of
thoracic malignancy often support the diagnosis of SVCS. Plain radiography is helpful and may
reveal a mediastinal mass in many patients. Symptoms most often include:

 Facial swelling and cyanosis

 Cough

 Chest discomfort

 Dyspnea

 Fever

 Weight loss

 Blurred vision

Signs most often include:

 Swelling of upper extremities and face

 Superficial vein dilatation

 Lymphadenopathy

 Hoarseness
 Horner’s syndrome

Chronic obstructive pulmonary disease or COPD (option B), is a mixture of 3 separate disease
processes. The patient has symptoms of this classic triad, which includes:

 chronic bronchitis

 emphysema

 asthma (but to a lesser extent)

Signs and symptoms of chronic bronchitis (blue bloaters):

 Patients may be obese

 Frequent productive cough

 Intermittent dyspnea

 Recurrent pulmonary infections

 Cardiac/respiratory failure over time, with edema and weight gain

 Coarse rhonchi and wheezing may be heard on auscultation

Signs and symptoms of emphysema (pink puffers):


 Patients may be very thin with a barrel chest.

 Little or no cough or expectoration

 Progressive dyspnea

 The chest may be hyperresonant, and wheezing may be heard; heart sounds are very
distant.

 Cachexia (wasting syndrome causing loss of weight, muscle atrophy, fatigue,


weakness)

 Respiratory failure

Congestive heart failure, CHF (choice A) may present as left sided, right sided or biventricular
failure. Patient with left sided failure usually have dyspnea on exertion or even at rest. In right
sided failure peripheral, pitting edema occurs. This does not match the scenario.

Pulmonary embolism (PE) (choice D) does not fit this clinical scenario.

The primary etiology of thrombosis of subclavian vein (choice E) is referred to as effort-


induced thrombosis or Paget-von Schrötter syndrome. It usually results from the excessive use
of the involved arm. External compression of the axillary-subclavian vein has been suggested
to contribute to the stasis of blood that leads to thrombosis. The most common lung
malignancy associated with subclavian thrombosis has been the Pancoast tumor which
presents with the following:

 Mild-to-moderate non-pitting edema

 Mild cyanosis of the hands and fingers on the affected side (whereas, bilateral swelling
in cases of SVCS)

 Dilatation of subcutaneous collateral veins may be present over the upper arm and
chest
 In a few cases, in which the diagnosis was missed or delayed or the patient presented
late, the thrombus may have extended to the superior vena cava. These patients show
most features of superior vena cava syndrome, including face and neck swelling,
periorbital edema, blurred vision, and some degree of facial cyanosis.
QUESTION A 2 year old child complaining of recurrent attacks of colicky
37 of 45
Questions
abdominal pain, recurrent non-bilious vomiting which
Notes subsequently became bilious and who also developed melena
and mucus discharge per rectum. On examination, there was a
sausage-shaped mass in right hypochondrium. The case was
diagnosed as intussusception.

Regarding intussusception, all of the following statements are


true except:

It is the commonest cause of


intestinal obstruction between
6 and 18 months

Usually occurs in the jejunum

The lead point may be a


Peyer's patch or Meckel's
diverticulum
Has a characteristic
'doughnut' appearance on
ultrasound
Red currant jelly is a Click on image to Zoom
characteristic sign

Explanation

The correct answer is B.


Intussusception is a process in which a loop of intestine invaginates into the lumen of an
adjacent intestinal loop.

In most cases it is idiopathic but it is thought to occur due to lymphoid tissue hypertrophy in
response to a rotavirus infection acting as a leading point for intussusception.

In 90% of cases it occurs at ileo-colic region as it has the highest concentration of lymph
nodes.

Answer A is a true phrase as most cases of intussusception is idiopathic and occurring at this
specific group of age.

Answer C is true phrase as lymph tissue of Peyer`s patch and is liable to hypertrophy with
infection acting as a leading point inducing vigorous peristalsis. Also the mouth of Meckel`s
diverticulum acts by the same mechanism.

Answer D is a true phrase. Doughnut (ring-like) appearance seen in longitudinal view of


ultrasonography is made by the multiple layers representing the walls of intussuscepted bowel
loops.

Answer E is a true phrase. Red currant jelly is formed by sloughed mucosa (mucosa of
intussuscepted loop become ishaemic and sloughed), blood and mucus.
QUESTION A 49-year-old woman comes to your office and complains of
38 of 45
Questions
significant pain in her lower extremities that develops every
Notes time she ambulates more than a few blocks (300 meters). This
has been ongoing for the past 5 months and seems to be
worsening with time. Her pain relieved after rest and
redevelops immediately after she continues to ambulate. Over
the last 3 days, she has noticed numbness that has developed
on her bilateral dorsal feet. On history, she is a smoker but
denies any history of diabetes or vascular disease. Her body
mass index is 32.
On physical examination, her lower extremities are cool and
pale, without swelling, tenderness or erythema. No dorsalis
pedis or posterior tibial pulses are palpable. Other systemic
findings are normal.

Which of the following abnormalities of the vasculature is most


likely to account for these findings?

Lymphatic obstruction

Arteriolosclerosis

Atherosclerosis

Medial calcific sclerosis

Venous thrombosis

Explanation

The correct answer option is C.

The main concept of the question is to assess your knowledge about arterial, venous and
lymphatic diseases of lower limb.

Arteriolosclerosis (an arterial disease) (choice B) is the hardening and loss of elasticity of small
arteries and arterioles. Types of arteriosclerosis include :
 Athersclerosis (also known as arteriosclerotic vascular disease or ASVD)

 Arteriosclerosis obliterans

 Medial calcific sclerosis (Monckeberg’s calcific sclerosis)

Atherosclerosis (also known as arteriosclerotic vascular disease or ASVD) (choice C) is a


condition in which an artery wall thickens as the result of a build-up of fatty materials such as
cholesterol. This is the leading cause of occlusive arterial disease of the lower extremities. The
site of claudication is distal to the location of the narrowed (stenotic) segment.

The most common presenting symptom is intermittent claudication as in this particular case.
The clinical term is used to describe muscle pains, aches, cramps, numbness or sense of
fatigue. These symptoms come and go with exertion and rest. With more severe disease, pain
or numbness of the foot occurs at night when in a non-dependent position. Symptoms improve
when the foot is placed in a dependent position. In severe claudication, the pain is also felt at
rest. Signs and symptomsof peripheral vascular disease include the following:

 decreased or absent distal pulses

 hair loss

 thickened nails

 shiny skin

 a skeletonized appearance

 pallor on elevation;

 rubor on dependency

 ulcers and gangrene


Finally, the progression of the disease is closely associated with cigarette smoking and
diabetes mellitus. Diabetic control and smoking cessation slows the progression of the disease.

Lymphatic obstruction (choice A) is a blockage of the lymph nodes and their vessels that drain
fluid from tissues throughout the body and allow immune cells to travel where they are
needed. Lymphatic obstruction is also called lymphedema, which means swelling of the lymph
passages. Lymph nodes may be enlarged for any reason. The main symptom is persistent
(chronic) swelling, usually of the arm or leg.

Arteriosclerosis obliterans (answer B) is typically seen in medium and large arteries of the
lower extremity and characterized by fibrosis of the intima and calcification of the media. The
lumen of the vessel may be obliterated or markedly narrowed. It is often associated with
hypertension and symptoms of Peripheral artery disease appear.

Medial calcific sclerosis (choice D) is a form of arteriosclerosis or vessel hardening, where


calcium deposits form in the middle layer of the walls of medium sized vessels. It does not
actually cause narrowing of the vessel lumen. In advanced cases, vessels may become rigid
and lose their distensibility. It is seen mostly in the elderly and diabetics, and commonly
involves the arteries of the thyroid and uterus.

Venous thrombosis (option E) is the development of a blood clot (thrombus) in a vein due to
reduced blood flow and abnormal coagulation. It usually begins with injury to the vein. The
process may occur in superficial or deep veins. DVT's produces no symptoms in 50% of
individuals; the other 50% may complain of swelling, tenderness, pain, skin discoloration and
warmth in the affected area.
QUESTION A neonate presented to the hospital with repeated bile-stained
39 of 45
Questions
vomiting, abdominal distension, and failure to pass meconium.
Notes Chest examination revealed associated bronchitis. The
diagnosis was made as suspected meconium ileus.

Regarding meconium ileus, all of the following statements are


true except:
Is a rare cause of neonatal
intestinal obstruction
many cases are associated
with cystic fibrosis
Obstruction usually occurs in
the distal ileum
Presents with neonatal bile-
stained vomiting and
abdominal distension
A plain x-ray may show an
intra-luminal 'soap bubble'
appearance

Click on image to Zoom

Explanation

The correct answer is choice A.

Meconium ileus accounts for approximately 33% of all cases of neonatal intestinal obstruction.
Other common causes of neonatal intestinal obstruction are jejuno-ileal atresia , dudenal
extrinsic and intrinsic obstruction.

Answer B is a correct phrase as meconium ileus is the early presentation of 20% of cases of
cystic fibrosis. Cystic fibrosis is an autosomal recessive disorder clinically characterized by the
following triad:

 Chronic obstruction and infection of respiratory tract.

 Exocrine pancreatic insufficiency (lack of pancreatic lipase that liquifies the


meconium).

 Elevated sweat chloride level.


Answer C is a correct phrase as usually at proximal part of ileum, the meconium is still fluidy
to some extent and then gradual insipissation occurs as it passes towards the terminal ileum.

Answer D is a correct phrase as meconium ileus causes intestinal obstruction which produces
abdominal distension and repeated vomiting that becomes bile-stained eventually.

Answer E is a correct phrase as the air become trapped inside the thick viscid meconium
giving a pathogonomonic soap bubble appearance.

QUESTION You are called to examine a neonate who was born with
40 of 45
Questions
excessive oral secretions. He has a sonorous cough and on
Notes breast feeding he seems to choke. On examination, there were
evident respiratory distress and crepitations heard all around
the chest. An orogastric tube is introduced but it was arrested.
Also you noticed that the baby has polydactyly and imperforate
anus. Your initial diagnosis was esophageal atresia with
tracheo-esophageal fistula.

Regarding esophageal atresia and tracheo-oesophageal fistula


(TOF), all of the following statements are true except:
Most cases of esophageal
atresia are associated with a
proximal TOF to the lower
oesophagus
Radiological evidence of gas
in the stomach confirms the
presence of distal TOF
Polyhydramnios is a common
feature in a fetus with
oesophageal atresia

The diagnosis my be
confirmed by the inability to
pass a nasogastric catheter
Oesophageal atresia with TOF
is a part of Vactrel syndrome Click on image to Zoom

Explanation

The correct answer is choice A.

85% of cases are associated with distal TOF and only less than 1% of cases are
associated with proximal TOF. Tracheo-oesophageal fistula develops due to
failure of fusion of tracheo-oesophageal folds. These folds normally fuses at 4th
to 6th week of gestation dividing the foregut into a ventral laryngo-tracheal tube
and a dorsal oesophageal tube. This leads to persistent abnormal communication
between trachea and oesophagus.

Choice B is a correct phrase as in presence of distal TOF, air passes through the
fistula to stomach causing gaseous distention.

Choice C is a correct phrase as polyhydramnios occurs in 33% of cases with


distal TOF and 100% of cases with absent TOF due to inability of the fetus to
effectively swallow amniotic fluid.

Choice D is a correct phrase as on introducing orogastric tube it is arrested at


the site of atresia characteristically 10 to 11 cm from the lips.

Choice E is a correct phrase as VACTREL syndrome includes vertebral defects,


anorectal malformation, cardiovascular disease, TOF, renal anomalies and limb
deformities.
QUESTION A 3 months old infant was brought to you complaining of
41 of 45
Questions
vomiting that started infrequently but now has become
Notes projectile and recurrent after every feeding. On examination,
the baby was malnourished and vigorous peristalsis is seen in
abdomen. An olive-like mass is palpated in upper right
quadrant. The case was diagnosed as hypertrophic pyloric
stenosis.

Regarding infantile hypertrophic pyloric stenosis, which of the


following statements is true:

Usually presents between 6


and 12 months of age

The male:female ratio is 4:1

Has a strong familial


predisposition
Pathologically shows Click on image to Zoom
hypertrophy of the
longitudinal muscle layer of
the pylorus
Presents with bile-stained
projectile vomiting
Explanation

The correct answer is C.

Hypertrophic pyloric stenosis is characterized by hypertrophy and hyperplasia of the circular


muscle fibers of the pylorus causing narrowing or even obstruction of the pyloric canal.

A familial predisposition is strongly suggested by the following :

 Occurs in 7% of infants of affected parents.

 Occurrence is reported in 3 successive generations of some families.

 85% coincidence in monozygotic twins.

 8% coincidence in dizygotic twins.

Answer A is a false phrase as 95% of cases are diagnosed within 3-12 months of age. Usually
repeated vomiting starts within 4 weeks of birth making the diagnosis most likely to occur very
early.

Answer B is a false phrase as female : male ratio is 5 : 1. Highest incidence occurs in first-
born male.

Answer D is a false phrase as the hypertrophy occurs in circular muscle layer not the
longitudinal layer.

Answer E is a false phrase as the vomiting is characteristically non-bilious as the obstruction


occurs prior to the duodenum into which the bile is secreted.
QUESTION A 46 year-old man complains of right upper quadrant pain and
42 of 45
Questions
has jaundice on clinical exam. On review of his labs, he has
Notes elevated alkaline phosphatase and increased direct bilirubin.
An ultrasound scan of the abdomen demonstrated a dilated
common bile duct. He proceeds to endoscopic retrograde
cholangio-pancreatography (ERCP) for retrieval of stone from
the common bile duct.

The staff doctor asks you where does the ampulla of Vater
enter the bowel?

Jejunum

Ascending (fourth part) duodenum

Descending (second part) duodenum

Inferior (third part) duodenum

terminal ileum

Explanation

The correct answer is choice C

The common bile duct (ductus choledochus) is a tube-like anatomic structure in the human
gastrointestinal tract. It is formed by the union of the common hepatic duct and the cystic duct
(from the gall bladder). It is later joined by the pancreatic duct to form the ampulla of Vater.
The ampulla of Vater, also known as the hepatopancreatic ampulla, is specifically located at
the major duodenal papilla. The Ampulla of Vater is an important landmark, halfway along the
second part of the duodenum, that marks the anatomical transition from foregut to midgut
The two ducts are surrounded by the muscular sphincter of Oddi. When the sphincter of Oddi
is closed, newly synthesized bile from the liver is forced into storage in the gall bladder. When
open, the stored and concentrated bile exits into the duodenum. This conduction of bile is the
main function of the common bile duct. The hormone cholecystokinin, when stimulated by a
fatty meal, promotes bile secretion by increased production of hepatic bile, contraction of the
gall bladder, and relaxation of the Sphincter of Oddi.

Several problems can arise within the common bile duct. While stones can frequently pass
through the common bile duct into the duodenum, some stones may be too large to pass
through the CBD and may cause an obstruction. One risk factor for this is duodenal
diverticulum. If clogged by a gallstone, a condition called choledocholithiasis can result. In this
clogged state, the duct is especially vulnerable to an infection called ascending cholangitis.
Very rare deformities of the common bile duct are cystic dilations (4 cm), choledochoceles
(cystic dilation of the ampula of Vater (3-8 cm)), and biliary atresia.

Choledocholithiasis causes jaundice and liver cell damage, and is a medical emergency. The
diagnosis of choledocholithiasis is suggested when the liver function blood test shows an
elevation in bilirubin. The diagnosis is confirmed either with an MRCP, ERCP, or an
intraoperative cholangiogram. If the patient must have the gallbladder removed for gallstones,
the surgeon may choose proceed with the surgery, and obtain a cholangiogram during the
surgery. If the cholangiogram shows stone in the bile duct, the surgeon may attempt to treat
the problem by flushing the stone into the intestine or retrieve the stone back through the
cystic duct.

Jejunum (choice A) lies between the duodenum and the ileum. The change from the
duodenum to the jejunum is usually defined as the ligament of Treitz.

Ascending (fourth part) duodenum (choice B) reaches the inferior border of the body of the
pancreas. Then, it curves anteriorly and terminates at the duodenojejunal flexure where it
joins the jejunum. The duodenojejunal flexure is surrounded by a peritoneal fold containing
muscle fibres: the ligament of Treitz. The amuplla of vater does not drain into the ascending
duodenum.

Inferior (third part) duodenum (choice D) begins at the inferior duodenal flexure and passes
transversely to the left, crossing the right ureter, right testicular/ovarian vessels, inferior vena
cava, abdominal aorta, superior mesenteric artery and the vertebral column. The amuplla of
vater does not drain into the inferior duodenum.
terminal ileum (choice E) is the most distal part of the small intestine. It connects to the
cecum, the pouch between the small and the large intestine, via the ileocecal valve. The
amuplla of vater does not drain into the terminal ileum.
QUESTION Predisposing factors increasing risk of infection after surgical
43 of 45
Questions
inervention are the following EXCEPT:
Notes

Mechanical ventilation use

Previous aminoglycoside therapy

Peritonitis

Laparoscopic technique

Longer hospital stay and APACHE II score on admission

Explanation

The correct answer is Choice D.

Nosocomial infections are major consideration in mortality rate in surgical intensive care units.
High incidence of resistance to multiple antibacterials and surgical complications contribute to
this rate. In the United States, about 2-5% of patients undergoing surgical procedure develop
surigical site infection (SSI). Overall SSI rate was found to be significantly lower for
laparoscopic technic for cholecystectomy (0.62%) than for the open alternative of gallbladder
surgery (1.82%). SSI risk was decreased in patients undergoing the laparoscopic technique
than with the open technique (Choice D).

Bloodstream infections and ventilator-assisted pneumonia were the most common type of
infections and increase mortality rate. The most common organisms identified in several
studies are the following:

 Acinetobacter baumannii

 Pseudomonas aeruginosa

 Candida albicans

 Enterococcus faecalis

 Klebsiella pneumoniae

 Enterococcus faecium

 Staphylococcus aureus

Multi-drug resistance to infection, of which previous aminoglycoside therapy contributes to,


can be linked to more severe medical comorbid conditions and contributes to the high
incidence of nosocomial infection (Choice B).

Due to complications of contamination, peritonitis and abdominal surgery increase the risk of
bloodstream infection and bacterial seeding within the abdominal cavity (Choice C).

The APACHE II or Acute Physiology and Chronic Health Evaluation II is a measure of severity
of disease classification system used as one of the scoring systems in the intesive care setting.
The higher the APACHE II score, the more severe the disease and the higher the mortality rate
(Choice E).
QUESTION A four day old infant delivered at home presents with bilious
44 of 45
Questions vomitting and abdominal distention. He has not passed meconium
Notes since birth. You suspect a case of Hirschsprung's disease.
All of the following statements are true except:

Often presents with neonatal


large bowel obstruction
Results from absence of
ganglion cells in both the
Meissner's and Auberbach's
plexus
A contrast-study will show
dilatation of the aganglionic
segment
Early treatment may involve Click on image to Zoom
rectal irrigation or an
emergency colostomy

Associated with trisomy 21

Explanation

The correct answer is Choice C

Hirschsprung’s disease is also known as aganglionic megacolon, a congenital


disease caused by a failure of parasympathetic ganglion cells to migrate into the
hindgut resulting in a functional obstruction of the large intestine. Both the
myenteric (Auerbach) and the submucosal (Meissner) plexus are absent (Choice
B) resulting in reduced bowel peristalsis. Absence of parasympathetic innervation
begins in the anus and extends proximally for a variable distance causing
functional obstruction at the junction (transition zone) between normal bowel and
the distal aganglionic bowel. In 80% of cases, the lesion is confined to the recto-
sigmoid colon (short-segment disease) and in 20% of cases, the entire colon is
involved (long-segment disease).
The incidence of Hirschsprung’s is 1 in 5000 live births, occurring three times
more commonly in males than females. The condition has been associated with
Down’s syndrome (trisomy 21) (Choice E) and multiple endocrine neoplasia type
II. There is a positive family history in 10% of cases. 99% of normal infants pass
meconium within 24 hours of birth, and Hirschsprungs disease usually presents
within the first few days of life with features of large bowel obstruction (Choice A)
including failure to pass meconium, abdominal distension and bile-stained
vomiting. The diagnosis is suspected clinically, but can usually be confirmed by;

 Plain abdominal x-rays; may show distended loops of bowel with reduced air in the
rectum

 Barium enema; contrast studies will show a constricted, NOT dilated distal
aganglionic segment (Choice C), a narrow transition zone and a dilated
proximal bowel segment

 Rectal biopsy; obtained under anaesthesia using punch or suction biopsy.


Positive biopsy will show the absence of ganglion cells confirmed by
staining for anticholinesterase

 Anorectal manometry; demonstrates absence on normal inhibitory reflex of


the internal sphincter on rectal lumen distension

Differential diagnosis includes anorectal malformations, intestinal atresia or


stenosis and intestinal motility disorders.

Conservative medical treatment is used to treat complications and institute


temporary measures until definitive reconstructive surgery is available. Colonic
lavage using mechanical irrigation may be required. Initial surgical treatment
includes creating a diverting colostomy at the time of diagnosis (Choice D).

Definitive surgery is usually performed when the child grows and weighs more
than 10 kg.
QUESTION You were called to examine a newborn infant that presented
45 of 45
Questions
with bilious vomiting, abdominal distension and failure to pass
Notes meconium. Double bubble sign was seen on a plain erect X-ray.

This picture may suggest the diagnosis of which of the


following:

congenital megacolon

Meckel's diverticulum

necrotizing enterocolitis

hypertrophic pyloric stenosis

duodenal atresia
Click on image to Zoom

Explanation

The correct answer is choice E.

Duodenal atresia is a congenital anomaly caused by failure of recanalization of the epithelium


of the duodenum. Also annular pancreas produce a similar condition as the pancreatic tissue
surrounds the entire circumference of the duodenum compressing it. In both cases, it is
presented by non-bilious vomiting as the obstruction occurs proximal to the ampulla of Vater,
failure of passage of meconium and abdominal distention. Double bubble sign appears in plain
erect x-ray films. The first bubble corresponds to the stomach and the second to the
postpyloric and prestenotic dilated duodenal loop.

Answer A is false. Congenital megacolon or Hirschsprung disease is caused by congenital


absence of ganglion cells in the distal alimentary tract. It is usually presented by bilious
vomiting and the x- ray show multiple loops of dilated small bowel with air-fluid levels that can
usually be determined to be a distal bowel obstruction.

Answer B is false as Meckel diverticulum is presented mainly by lower GI bleeding. Intestinal


obstruction occurs in complicated cases only and in such case the vomiting is bilious and the
double bubble sign is absent.

Answer C is false. Necrotizing enterocolitis is not presented by double bubble sign. The main
presentation includes abdominal distension, frank or occult blood in the stools vomiting,
diarrhea, feeding intolerance and high gastric residuals following feedings.

Answer D is false as Hypertrophic pyloric stenosis is presented by a single bubble sign as the
obstruction in this case occurs at the pylorus, proximal to the duodenum.

Das könnte Ihnen auch gefallen